krok 2 - 2011 question paper (stomatology)

28
Kok 3 Stomatology 2011 1 1. A 38-year-old patient complains of tension and enlargement of her lower lip and eyelids occuring after taking aspirin for headache. The edema resolves soon after taking antihistaminic drugs. What is the most likely diagnosis? A. Quincke’s edema B. Melkersson-Rosenthal syndrome C. Erysipelas of lips D. Lymphadenic macrocheilitis E. Meige trophedema 2. Parents of a 5-month-old baby complain of food refusal, ulcers on the palate. The infant was born prematurely, is now arti- cially fed. Objectively: at the junction of hard and soft palate there is an oval well- dened ulcer, covered with yellow-gray lm and limited by a hyperemic swelli- ng, protruding above the surface of oral mucosa. Which group of drugs should be administered for the aphtha epithelizati- on? A. Keratoplastic agents B. Antiviral drugs C. Antimycotic drugs D. Antiseptics E. Antibiotics 3. A 35-year-old male consulted a denti- st about the 26 tooth extraction. After the patient had been given tuberal anaesthesia, he presented with progressi- ng tissue edema and limited mouth openi- ng. This condition was caused by: A. Vessel trauma B. Muscle trauma during anaesthetization C. Nerve trunk trauma D. Hypersensitivity to the anaesthetic E. Anaphylactic shock 4. After removal of dental deposit an 18- year-old patient underwent preventive examination. It revealed painless chalky spots in the precervical area on the vesti- bular surface of the 22 and 41 teeth. Result of enamel resistance test is 7. What morphological changes are typical for this disease? A. Subsurface enamel demineralization B. Changes in the mantle dentine C. Damage of dentinoenamel junction D. Supercial enamel demineralization E. Degeneratic changes of odontoblasts 5. A 10-year-old child complains of sore throat, cough, fever (up to 38 o C ). These presentations turned up 2 days ago. Objectively: acute catarrhal stomatitis is present. Tonsils are swollen, hyperemic, covered with yellow-gray friable lm whi- ch can be easily removed. Submandi- bular and cervical lymph nodes are si- gnicantly enlarged, painful on palpation. Laboratory analysis revealed leuko- and monocytosis. What is the most likely di- agnosis? A. Infectious mononucleosis B. Diphtheria C. Scarlet fever D. Rubella E. Measles 6. A 50-year-old patient complains about missing of the 37, 36, 45, 46, 47 teeth. The 35, 38, 44, 48 teeth are intact and stable, their crowns exhibit anatomic form, sufcient height, relative paralleli- sm. Fulcrum line typically goes through the 35 and 44 abutment teeth. What xing element should be used in a clasp denture supported by the 35, 44 teeth? A. Acker clasp B. Roach clasp C. Jackson clasp D. Telescopic xation E. Bent wire clasp 7. A 60-year-old patient complains of pain in the masticatory muscles and temporomandibular joint. The dentures were fabricated a month ago. Objecti- vely: face conguration is abnormal, the lower third of the face is elongated, the li- ps can only be closed with difculty, the speech is indistinct. When smiling, the base of the complete removable denture is exposed. What mistakes were made duri- ng the fabrication of complete removable dentures for the upper jaw? A. The vertical occlusal dimension was overrated B. The vertical occlusal dimension was underrated C. Misidentied posterior occlusion D. Misidentied anterior occlusion E. Misplacement of maxillary teeth 8. A 58-year-old patient needs dental prosthetics. Objectively: the 22 tooth is stable, intact. The atrophy of the alveolar crest is present, the palate is at. It is planned to fabricate a removable denture. What tactics should be chosen in respect of the 22 tooth?

Upload: eneutron

Post on 15-Apr-2017

393 views

Category:

Health & Medicine


12 download

TRANSCRIPT

Page 1: Krok 2 - 2011 Question Paper (Stomatology)

Kok 3 Stomatology 2011 1

1. A 38-year-old patient complains oftension and enlargement of her lower lipand eyelids occuring after taking aspirinfor headache. The edema resolves soonafter taking antihistaminic drugs. What isthe most likely diagnosis?

A. Quincke’s edemaB. Melkersson-Rosenthal syndromeC. Erysipelas of lipsD. Lymphadenic macrocheilitisE. Meige trophedema

2. Parents of a 5-month-old baby complainof food refusal, ulcers on the palate. Theinfant was born prematurely, is now arti-ficially fed. Objectively: at the junction ofhard and soft palate there is an oval well-defined ulcer, covered with yellow-grayfilm and limited by a hyperemic swelli-ng, protruding above the surface of oralmucosa. Which group of drugs should beadministered for the aphtha epithelizati-on?

A. Keratoplastic agentsB. Antiviral drugsC. Antimycotic drugsD. AntisepticsE. Antibiotics

3. A 35-year-old male consulted a denti-st about the 26 tooth extraction. Afterthe patient had been given tuberalanaesthesia, he presented with progressi-ng tissue edema and limited mouth openi-ng. This condition was caused by:

A. Vessel traumaB. Muscle trauma during anaesthetizationC. Nerve trunk traumaD. Hypersensitivity to the anaestheticE. Anaphylactic shock

4. After removal of dental deposit an 18-year-old patient underwent preventiveexamination. It revealed painless chalkyspots in the precervical area on the vesti-bular surface of the 22 and 41 teeth.Result of enamel resistance test is 7. Whatmorphological changes are typical for thisdisease?

A. Subsurface enamel demineralizationB. Changes in the mantle dentineC. Damage of dentinoenamel junctionD. Superficial enamel demineralizationE. Degeneratic changes of odontoblasts

5. A 10-year-old child complains of sorethroat, cough, fever (up to 38oC). Thesepresentations turned up 2 days ago.Objectively: acute catarrhal stomatitis is

present. Tonsils are swollen, hyperemic,covered with yellow-gray friable film whi-ch can be easily removed. Submandi-bular and cervical lymph nodes are si-gnificantly enlarged, painful on palpation.Laboratory analysis revealed leuko- andmonocytosis. What is the most likely di-agnosis?

A. Infectious mononucleosisB. DiphtheriaC. Scarlet feverD. RubellaE. Measles

6. A 50-year-old patient complains aboutmissing of the 37, 36, 45, 46, 47 teeth.The 35, 38, 44, 48 teeth are intact andstable, their crowns exhibit anatomicform, sufficient height, relative paralleli-sm. Fulcrum line typically goes throughthe 35 and 44 abutment teeth. What fixingelement should be used in a clasp denturesupported by the 35, 44 teeth?

A. Acker claspB. Roach claspC. Jackson claspD. Telescopic fixationE. Bent wire clasp

7. A 60-year-old patient complains ofpain in the masticatory muscles andtemporomandibular joint. The dentureswere fabricated a month ago. Objecti-vely: face configuration is abnormal, thelower third of the face is elongated, the li-ps can only be closed with difficulty, thespeech is indistinct. When smiling, thebase of the complete removable denture isexposed. What mistakes were made duri-ng the fabrication of complete removabledentures for the upper jaw?

A. The vertical occlusal dimension wasoverratedB. The vertical occlusal dimension wasunderratedC. Misidentified posterior occlusionD. Misidentified anterior occlusionE. Misplacement of maxillary teeth

8. A 58-year-old patient needs dentalprosthetics. Objectively: the 22 tooth isstable, intact. The atrophy of the alveolarcrest is present, the palate is flat. It isplanned to fabricate a removable denture.What tactics should be chosen in respectof the 22 tooth?

Page 2: Krok 2 - 2011 Question Paper (Stomatology)

Kok 3 Stomatology 2011 2

A. It is required to fabricate a telescopiccrownB. There is no need of any interventionC. It should be depulpedD. It should be extractedE. It is required to fabricate a stump crown

9. A 39-year-old patient needs a claspdenture with porcelain teeth for the lowerjaw. What method should be applied whi-le substituting wax with plastic in orderto avoid mold flash that may cause biteopening?

A. Method of transfer moldingB. Direct method of compression moldingC. Inverse method of compression moldingD. Combined method of compressionmoldingE. In water under pressure

10. A 22-year-old patient was undergoingtreatment of the 25th tooth for deep acutecaries. During preparation a dentist acci-dentally opened the horn of pulp. Whatmethod of treatment should be applied?

A. Biologic methodB. Vital amputation of pulpC. Devital amputationD. Vital extirpationE. Devital extirpation

11. A 39-year-old patient complains ofsome soft ulcers and tubercles on the oralmucosa, gingival haemorrhage, pain andloosening of teeth. Objectively: mucousmembrane of tongue and gums presentssingle ulcers with soft, swollen, slightlypainful edges, covered with a yellow film.Regional lymph nodes are enlarged, soft,painless, not adherent to the surroundi-ng tissues. What is your provisional di-agnosis?

A. Lupus tuberculosisB. LepraC. Tertiary syphilisD. ScrofulodermaE. Sutton’s aphthae

12. A patient complains about mobili-ty of the 26, 27, 28 teeth, periodic pain,sensation of heaviness in the correspondi-ng half of the upper jaw, haemorrhagic di-scharges from the left side of nose. Thesepresentations have been observed for 4months, within the last month the painhas become worse, the patient presentsnow with general weakness. Examinati-on of the oral cavity reveals the gradeII mobility of the 26, 27, 28 teeth. Onpalpation of the maxillary tuber, the bone

wall was found to be absent. X-ray pi-cture shows a homogeneous shadowingof maxillary sinus; bone structures haveunclear contours. What is the most likelydiagnosis?

A. Upper jaw cancerB. Upper jaw adamantinomaC. Upper jaw sarcomaD. Chronic odontogenous maxillary sinusi-tisE. Upper jaw osteoclastoma

13. An 8-year-old boy was diagnosed withchronic fibrous pulpitis of the 21 tooth.It was treated by extirpation method.Choose the material for root filling:

A. Calcium-containing materialB. Glass-ionomer cementC. Zinc oxide eugenol pasteD. Resorcin-formalin pasteE. Phosphate cement

14. A 62-year-old patient has a medi-an mandibular fracture complicated bypseudoarthrosis. Objectively: the dentalformula is 33, 34, 35, 36, 37, 47, 46, 45,44, 43. The teeth are intact, stable, withhigh crowns. There is minor mobility offragments without displacement. What isthe most appropriate type of denture forthis patient?

A. Dental bridge without a pivot pointB. Lamellar denture without a pivot pointC. Lamellar denture with Gavrilow’s pivotpointD. Lamellar denture with Oksman’s pivotpointE. Lamellar denture with Weinstein’s pivotpoint

15. During extraction of the 47 tooth itsdistal root was broken halfway along itslength. What tool should be chosen forextraction of the residual root fragments?

A. Left angled elevatorB. Broad-beaked forcepsC. Close-beaked forcepsD. Right angled elevatorE. Straight elevator

16. A patient consulted a doctor aboutrecurrent pain in the frontal part of hisupper jaw. He has history of previoustreatment for pulpitis. Objectively: thecrown of the 12 tooth is restored witha filling. X-ray picture of the upper jawshows an area of bone tissue destruction(up to 1 cm in diameter) at the root apexof the 12 tooth. The root channel of the

Page 3: Krok 2 - 2011 Question Paper (Stomatology)

Kok 3 Stomatology 2011 3

12 tooth is filled up to the top. The pati-ent was diagnosed with cystogranuloma ofthe 12 tooth. Choose the best method oftreatment:

A. Root apex resectionB. Extraction of the 12 toothC. Conservative treatmentD. Dissection along the mucogingivaljunctionE. Replantation of the 12 tooth

17. Preventive examination of a 20-year-old student revealed chronic candidi-asis of the oral mucosa, generalizedlymphadenopathy. He has a 1-year historyof herpes simplex. The body temperatureoften rises to 37, 4 − 37, 5oC, the patienthas lost 8 kg of body weight over the lastmonth. These symptoms may be indicati-ve of the following disease:

A. AIDSB. Infectious mononucleosisC. Acute leukosisD. Chronic leukosisE. Lymphogranulomatosis

18. A 38-year-old male patient complai-ns of a carious cavity. He had experi-enced spontaneous dull pain in the toothin question before. Objectively: the di-stal masticatory surface of the 37 toothpresents a deep cavity made of soft pi-gmented dentin. Percussion is painless.After removing the decay from the cavi-ty, cold water has caused pain lasting forabout a minute. X-ray picture shows thedeformation of the periodontal gap in theregion of the 37 root apices. What is themost likely diagnosis?

A. Chronic fibrous pulpitisB. Exacerbation of chronic pulpitisC. Acute deep cariesD. Chronic deep cariesE. Chronic fibrous periodontitis

19. A 32-year-old patient complains ofexperiencing dryness and burning oftongue back for a week. The burningintensifies during taking irritating foods.Some time ago he had pneumonia andwas treated with antibiotics. Objectively:skin and oral mucosa are pale. The lingualmucosa is hyperemic and edematous, thefolds at the back of tongue are coveredwith the crumbling gray-white plaque,desquamation of the lateral surface oftongue is also present. Saliva is thickand attaches to a spatula. Choose themost effective drug complex for the localtreatment:

A. Methylene blue + pimafucinB. Hydrocortizone ointment + solcoserylC. Decamin + hydrocortizone ointmentD. Furacilin + methylene blueE. Furacilin + solcoseryl

20. A 67-year-old patient needs a completeremovable denture for his lower jaw. Theadjustment of custom tray is done bymethod of Herbst. It is required to speci-fy the boundary of tray for the sublingualregion, 1 cm away from the midline. WhatHerbst test should be applied for thispurpose?

A. Alternate touching the cheeks with thetongueB. Touching the red border of the upper lipwith the toungueC. Active movements of mimic muscledD. SwallowingE. Slow mouth opening

21. A 40-year-old patient has an openocclusion resulting from the bilateralfracture of his upper jaw and followingmalunion of fragments. The occlusal verti-cal dimension is maintained only by molarteeth. There is a gap of about 1-1,5 mmbetween the rest of teeth. The dentitionsare without gaps. What kind of treatmentwould be most appropriate in this situati-on?

A. The molar teeth should be ground offand covered with crownsB. Orthodontic treatmentC. Combined apparatus and surgicaltreatmentD. Open repositionE. Prosthetics with combined crowns

22. A 14-year-old child complains ofbleeding and pain during eating in thelower right molar. Objectively: the medialcontact and masticatory surfaces of the46 tooth exhibit a large carious cavityfilled with red tissue. Superficial probi-ng causes pain and moderate bleedi-ng. Deeper probing causes acute pain.Percussion is painlessl. What is your provi-sional diagnosis:

A. Chronic hypertrophic pulpitisB. Chronic fibrous pulpitisC. Chronic gangrenous pulpitisD. Chronic papillitisE. Chronic granulating periodontitis

23. A 10-year-old girl complains of mi-nor pain in a lower jaw tooth occuringduring having cold food. Objectively: the

Page 4: Krok 2 - 2011 Question Paper (Stomatology)

Kok 3 Stomatology 2011 4

masticatory surface of the 36 tooth exhi-bits a carious cavity with a narrow inletlocated within the mantle dentin. Probi-ng causes pain along the enamel-dentinborder. Make a diagnosis:

A. Acute median cariesB. Acute deep cariesC. Chronic deep cariesD. Superficial cariesE. Chronic median caries

24. An 80-year-old patient presentedto a clinic for the adjustment of theimpression tray for the upper jaw. Objecti-vely: alveolar process exhibits markedhomogenous atrophy, maxillary tubers areabsent, torus is weakly expressed. Buccalmuscles and frenulum of the upper lip areattached almost at the top of the alveolarprocess, the palatal slope is not steep.How to adjust the impression tray in orderto make a close-fitting valve?

A. To extend the borders along the A lineB. To extend the external bordersC. To extend the borders in the frontalregionD. To extend the borders in the region ofmaxillary tubersE. To overlay buccal muscles and frenulumwith the impression tray

25. A 32-year-old male complains ofconstant mandibular movements simulati-ng chewing that take place when he isexcited. Besides that, the patient complai-ns of teeth grinding. Objectively: thelateral surfaces of teeth have signs ofabrasion, the tubercles are absent. Whatis the most rational method of treatment?

A. Night guard, myogymnastics, self-massageB. Restoration with opposite-directeddenturesC. Medicamental and machine treatmentwith further consultation with a dentalsurgeonD. Consultation with a dental surgeonE. Teeth alignment (selective tubercleremoval)

26. An 18-year-old patient complainsabout body temperature rise, weakness,pain induced by eating and deglutition.Objectively: mucous membrane of theoral cavity is erythematic with multi-ple petechia. Pharynx is hyperaemic.Regional lymph nodes are enlarged,mobile, painless. In blood: leukocytosis,monocytosis, atypic mononuclears, ESR

is 30 mm/h. What is the leading factor ofdisease development?

A. Viral infectionB. Bacterial infectionC. Autoimmune disordersD. Immediate allergyE. Delayed allergy

27. A 72-year-old patient complains ofburning pain in the corners of mouth.Objectively: the folds of mouth cornershave erosions, covered with white coati-ng that can be easily removed, mucousmembrane of mouth corners is macerated,of pearly colour. There is pathologicaltooth wear and decreased vertical di-mension of occlusion. What is your provi-sional diagnosis?

A. Angular cheilitis candidiasisB. Atopic cheilitisC. Chronic recurrent labial fissureD. Streptococcal angular cheilitisE. Syphilitic angular cheilitis

28. A 45-year-old patient presentedto aclinic for prosthetics. Objectively:Kennedy III type, I subtype dentitiondefect is present, the 46, 45, 44, 34, 35,teeth are missing. The bite is fixed. CheCrowns of the 47, 43, 33, 37 teeth are high,of a well-defined anatomical shape, intact.It is planned to fabricate a clasp denture.What system of the clasp denture fixationis the most appropriate in this case?

A. Rumpel’s bar systemB. AttachmentsC. Roach claspD. Continuous claspE. Telescopic system

29. After a fall a 28-year-old patient visi-ted the clinic of maxillofacial surgery andcomplained of restricted mouth opening,skin numbness in the left infraorbital regi-on, upper lip, wing of the nose, pain in theleft zygomatic region that gets worse whenhe opens his mouth. Palpation revealedstep deformity. What is the most likely di-agnosis?

A. Fracture of the left zygomatic bone witha displacementB. Fracture of the left zygomatic bonewithout a displacementC. Fracture of the upper jaw of Le Fort II(medium type)D. Fracture of the upper jaw of Le Fort I(top type)E. Fracture of the left zygomatic bone

Page 5: Krok 2 - 2011 Question Paper (Stomatology)

Kok 3 Stomatology 2011 5

30. A 75-year-old patient has been admi-tted to the dental department for theright-angle mandibular fracture with adisplacement. Objectively: the face isasymmetric due to the post-traumaticswelling, palpation reveals the mobility offragments and pain. The patient has a li-felong history of epilepsy. What is the bestmethod of treatment?

A. OsteosynthesisB. Immobilization with Vasilyev splintsC. Immobilization with Tigerstedt splintsD. Immobilization with Port splintE. Immobilization with Vankevich splint

31. Analysis of a 10-year-old boy’s jawmodels revealed that occlusal plane ofthe frontal maxillary teeth was of concaveform, its lateral parts were convex. Formof the alveolar process also representsdeformation of dental arches. The upperjaw is of saddle-like form with abruptnarrowing in the region of premolar teeth.What type of bite is it?

A. OpenB. DistalC. DeepD. MesialE. Cross

32. A 59-year-old patient has a bilateralfracture of mandible in the region of the44, 34 teeth, other masticatory teeth aremissing, toothless fragments are not di-splaced but mobile. Which orthosis shouldbe used for the immobilization of bonefragments?

A. Vankevich splint with orthodonticelasticsB. Rudko’s apparatusC. Limberg’s splintD. Petrosov’s apparatusE. Zbarzh apparatus

33. A 23-year-old patient complains of gi-ngival haemorrhages, fatigue, indisposi-tion. The symptoms have appeared qui-te recently. Objectively: the skin andoral mucosa are pale. Gums bleed whentouched. There are petechiae on themucous membrane of cheeks, lips andpalate. What laboratory test will allow tomake a diagnosis?

A. Complete blood countB. Blood test for sugarC. Blood test for Vitamin CD. Bleeding time testE. Immunogram

34. A 20-year-old patient complains ofgeneral weakness, fever, headache. Thesepresentations appeared three days ago.Objectively: the regional lymph nodesare enlarged, painful on palpationl, bodytemperature is 37, 8oC, oral mucosais hyperemic and edematous. Mucousmebrane of lips, palate, gums, cheekspresents single erosions of polycyclicirregular shape, covered with grayish-white film, painful. Which drugs shouldbe administered for the local treatment ofthe early disease?

A. AntiviralB. Keratoplastic agentsC. Corticosteroid ointmentsD. AntisepticsE. Dyes

35. A patient has been diagnosed wi-th traumatic open angular fracture ofmandible with a fragments displacement.Which of the following should be appli-ed for transport immobilization of thefragments?

A. Entin head-chin strapB. Tigerstedt splintsC. Standard Vasyliev splintsD. Gunning splintsE. Weber splints

36. A 50-year-old patient needs dentalprosthetics. Objectively: the 14, 15, 16,17, 24, 25, 26, 27 teeth are missing. Thestate of the abutment teeth is satisfactory.What type of fixation will provide thebest conditions for the stabilization of aremovable denture?

A. PlanarB. SagittalC. TransversalD. LinearE. Point

37. A 6-year-old boy hit his forehead oneday ago. A few hours later a swellingappeared in the right superciliary regi-on. Objectively: there is a considerableedema of forehead tissues spreading toeyelids of the right eye, the skin over theswelling is cyanotic, the swelling is of softconsistency. Fluctuation is also present.General condition of the boy is normal.Make a provisional diagnosis:

Page 6: Krok 2 - 2011 Question Paper (Stomatology)

Kok 3 Stomatology 2011 6

A. Hematoma of the right superciliaryregionB. Postraumatic edema of tissues of theright superciliary regionC. Fracture of frontal boneD. Hematic abscess of the right supercili-ary regionE. Inflammatory infiltration of tissues ofthe right superciliary region

38. A 14-year-old child complains about acosmetic defect in the frontal teeth regi-on. Objectively: enamel of the 11, 12,21, 22, 31, 32, 41, 42 teeth is thin in theregion of cutting edge, there is a sulcateenamel pit 1,5 mm wide which encirclesthe tooth and is parallel to the cuttingedge. The cusps of the 16, 26, 36, 46 teethare underdeveloped and have conicalform. What is the most likely diagnosis?

A. Systemic hypoplasiaB. Dentinogenesis imperfectaC. Enamel dysplasiaD. Local hypoplasiaE. Stainton-Capdepont dysplasia

39. Preventive examination of a 45-year-old patient revealed a carious cavity inthe 23 tooth located within the mantledentin. The cavity floor and walls areof dense, markedly pigmented dentin,probing and cold test provoke no pain,percussion of the 23 tooth is painless.Electroodontometry results - 6 mA. Whatis the most likely diagnosis?

A. Chronic median cariesB. Acute median cariesC. Chronic deep cariesD. Acute deep cariesE. Chronic periodontitis

40. A 29-year-old patient undergoes aprocedure of a metal inlay fabrication bythe direct method. The inlay is intendedfor the 26 tooth. Objectively: the 26 toothpresents a Black class I carious cavi-ty. Electroodontometry results - 6 mA.Examination of the ready inlay revealed acasting defect in the region of the externalangle. The dentist’s further actions will be:

A. Fabrication of a new inlayB. Cement fixation of the inlayC. Plastic fixation of the inlayD. Inlay fixation and restoration of thedefect with a compositeE. Inlay fixation and restoration of thedefect with amalgam

41. A 53-year-old patient complains of mi-ssing of mandibular teeth. He has a hi-

story of microstomy. Objectively: the 31,32, 35, 36, 37, 38, 41, 42, 45, 46, 47 and 48teeth are missing. The remaining teeth arestable, immobile. What kind of dentureconstruction is recommended for this pati-ent?

A. Collapsible removable partial lamellardentureB. Removable partial lamellar dentureC. Dental bridge supported by the 43, 33teeth and removable dentureD. Clasp dentureE. Splint-denture

42. During the examinations and everydayorthopedic manipulations a dentist uses adental mirror. How to sterilize dental mi-rrors?

A. In the triple solution for 30 minutesB. In the 0,5% ethyl chloride solution for20 minutesC. In the 6% hydrogen peroxide solutionfor 6 hoursD. In the dry heat sterilizer at 180oC for 10minutesE. in the 0,01% chloramine solution for 10minutes

43. A 35-year-old woman was admi-tetd to the oral surgery department twohours after a road accident. Objectively:cutaneous integuments are pale, there arebeads of sweat on her face. Respirati-on is heavy and obstructed. Pulse rateis 120 bmp, AP is 70/60 mm Hg. Thereis wound penetrating to the oral cavityon the left cheek. During transportationthe woman was in supine position withher head thrown backwards. What kind ofasphyxia is likely to develop in this pati-ent?

A. AspiratingB. ValvularC. StenoticD. ObturativeE. Dislocational

44. A 10-year-old girl complains of anaesthetic defect. She has a history ofsucking her right thumb till the age of 7.Objectively: the height of the lower thirdof face is somewhat reduced. There is a 9mm gap in sagittal direction between theupper and lower incisors, Engle’s class 2.As a result of Eschler-Bittner test the gi-rl’s face appears at first better, then worse.What clinical form of occlusal anomaly ismost likely?

Page 7: Krok 2 - 2011 Question Paper (Stomatology)

Kok 3 Stomatology 2011 7

A. Maxillary macrognathia and mandi-bular micrognathiaB. Maxillary macrognathiaC. Mandibular micrognathiaD. Maxillary prognathism with lateralcompressionE. Mandibular retrognathia

45. A 36-year-old patient requires a fullcoverage swaged metal crown for the37 tooth. The patient presented to a cli-nic for the crown adjustment. Duringthe adjustment it was revealed that thecrown was to loose to fit the tooth cervixproperly. How to eliminate this defect?

A. To get an impression and swage thecrown anewB. To swage a new crown using the samematrixC. To bend in the crown edges by usingbeak-shaped forcepsD. To anneal the crownE. To bend in the crown edges by usingclamp forceps

46. A 31-year-old patient complains ofexperiencing recurrent pain in the 48tooth for 4 years. Objectively: rightsubmandibular lymph nodes are enlarged,painless. Mucous membrane around thepartialy erupted 48 tooth is hyperemic. Onprobing the subgingival pocket of the 48tooth exudes pus mixed with blood. Whatadditional study will be most informativefor diagnosing this pathology?

A. RadiographyB. Blood countC. ElectroodontometryD. Blood and urine testsE. Contrast radiography

47. A 44-year-old patient consulted asurgeon about constant acute pain in theupper jaw region on the left that is getti-ng worse during teeth joining. The painwas noted 3 days ago. Objectively: theface is symmetrical, mouth opening isunlimited. The crown fof the 26 toothis half-destroyed. Probing of the cari-ous cavity is painless. Percussion of the26 tooth provokes acute pain. Mucousmembrane of the alveolar process isedematic, hyperemic on the level of the26 tooth. The 26 tooth was treated before.What is your provisional diagnosis?

A. Exacerbation of chronic periodontitisof the 26 toothB. Acute purulent periodontitis of the 26toothC. Acute pulpitis of the 26 toothD. Acute purulent periostitis of upper jawof the 26 tooth on the leftE. Periodontitis of the 26, 27, 28 teeth

48. A 38-year-old patient complains ofacute paroxysmal pain in the region ofhis left upper jaw, left eye and temple. Thepain is lasting (2-3 hours), gets worse atnight. Objectively: the 26 tooth has a deepcarious cavity, floor probing causes painfulresponse, thermal stimuli provoke long-lasting pain, percussion provokes minorpain. What is the most likely diagnosis?

A. Acute diffuse pulpitisB. Pulpitis complicated by the periodonti-tisC. Acute limited pulpitisD. Exacerbation of the chronic pulpitisE. Acute purulent pulpitis

49. A 46-year-old patient has been di-agnosed with chronic concrementouspulpitis of the 27 tooth. X-ray pictureshows that the concrement is located inthe crown part of the tooth cavity andoccupies 1/4 of its volume. Select anadequate method of treatment:

A. Vital extirpationB. Biologic methodC. Devital extirpationD. Vital amputationE. Devital amputation

50. The child is 13 years old. The diseasebegan suddenly with a body temperaturerise up to 39oC, swelling of the upperjaw on the left. Objectively: the faceis asymmetric due to the left cheekedema. The skin over the swollen area ishyperemic and tense. The 26 tooth exhibi-ts a large carious cavity, percussion of the26 tooth causes pain reaction. The 25, 26,27 teeth are mobile. Mucogingival juncti-on in the region of the 24, 25, 26, 27 toothis indistinct, alveolar mucosa is hyperemicand bilaterally swollen in the region of the25, 26. 27 teeth. What is the most likelydiagnosis?

Page 8: Krok 2 - 2011 Question Paper (Stomatology)

Kok 3 Stomatology 2011 8

A. Odontogenous maxillary osteomyelitisto the left of the 26 toothB. Odontogenous maxillary periostitis tothe left of the 26 toothC. Chronic periodontitis of the 26 toothD. Suppuration of the radicular cystinduced by the 26 toothE. Odontogenous purulent maxillarysinusitis induced by the 26 tooth

51. A 5-year-old child presents withchronic deep caries within the contactand masticatory surfaces of the 75, 74, 84,85 teeth. Which filling material should beused?

A. Silver amalgamB. SilidontC. EvicrolD. SilicinE. Infantid

52. A patient complains of generalweakness, pain in the gums, halitosis.Objectively: the patient is pale, adynamic,body temperature is 38, 5oC, submandi-bular lymph nodes are enlarged, pai-nful on palpation. Interdental gingivalpapillae are inflamed, their tops are"detruncated", covered with gray-yellownecrotic incrustation. Radiography ofalveolar process and blood count revealno apparent changes. What is the most li-kely diagnosis?

A. Acute ulcerous gingivitisB. Generalized periodontitisC. Acute leukosisD. AgranulocytosisE. Localized periodontitis

53. Two years after radiation therapy a49-year-old patient has been diagnosedwith recurrence of the lower lip carci-noma. Objectively: there is a 1x2 cm largeneoplasm with an ulcer in the centre in theregion of the right half of the lower lip.Palpation of the right submandibular regi-on reveals two enlarged, dense, painlesslymph nodes of round shape. Specify thestage of the disease:

A. T2 N1 M0B. T2 N0 M0C. T1 N1 M0D. T1 N2 M0E. T1 N0 M0

54. A 38-year-old patient ordered aclasp upper jaw prosthesis. Its fabricati-on involves forming of a fireproof model.What material should be applied forduplication?

A. GelinB. StomatoplastC. StomalginD. OrthocorE. Dentafol

55. A 20-year-old patient complains aboutunaesthetic look of the 24 tooth fromthe moment of its eruption. Objectively:enamel of the 24 tooth is partly absent,the dentine is yellow. The 64 tooth wastreated more than once when she was achild, but because of frequent exacerbati-ons, edemas and gingival fistula the 64tooth was extracted when she was 9 yearsold. What is the most likely diagnosis?

A. Localised hypoplasiaB. Initial cariesC. FluorosisD. Systemic hypoplasiaE. Enamel aplasia

56. A 45-year-old patient complains ofburning tongue, a metallic taste in mouth.Three months ago she got a dental bridgemade of gold and supported by the 16, 14teeth. Oral cavity exasmination reveals noobjective changes. The 36, 37, 46 teeth aresealed with amalgam fillings. What is themost likely cause of this condition?

A. Galvanic currentsB. AllergyC. Neurologic disorderD. Chemical factorsE. Mechanic trauma

57. A 25-year-old patient complains ofgingival haemorrhage and enlargement.Objectively: hypertrophy of gingival papi-llae by 1/3 of the crown height in thefrontal region of mandible. Periodontalpockets are absent. What is the most li-kely diagnosis?

A. Hypertrophic gingivitisB. Catarrhal gingivitisC. Ulcerous gingivitisD. Localized periodontitisE. Epulis

58. A chemical plant worker consulted adentist about an oral mucosa burn causedby caustic soda. Which of the followi-ng medications should be chosen foremergency care?

Page 9: Krok 2 - 2011 Question Paper (Stomatology)

Kok 3 Stomatology 2011 9

A. 0,5% acetic acid solutionB. 50% ethyl alcoholC. 0,1% liquid ammoniaD. 3% sodium chloride solutionE. 2% citric acid solution

59. A patient undergoes orthopaedictreatment of bounded edentulous spaceson both jaws by means of fullcast combi-ned dental bridges. The patient has a hi-story of essential hypertension. Whichhaemostatic drug SHOULD NOT be usedfor the gums retraction?

A. NoradrenalineB. Hydrogen peroxideC. Aluminium chlorideD. Iron sulphateE. Tetrahydrozoline

60. Parents of a 1,5-month-old chi-ld complain of whitish depositions onthe child’s buccal and labial mucosa.Objectively: labial and buccal mucosais hyperemic and covered with caseousdeposition that can be easily removed.Specify the causative agent of this disease:

A. Candida fungiB. Vincent’s spirochaetesC. Fusiform bacillusD. Loeffler’s bacillusE. Diplococci

61. A 7-year-old boy complains of feverup to 38oC, headache, sore throat.Objectively: slightly hyperemic soft palatemucosa, anterior palatine arches andtonsils exhibit erosions. Submandibularlymph nodes are slightly enlarged, pai-nless. What is the causative agent of thisdisease?

A. Coxsackie virusB. Herpes simplexC. Epstein-Barr virusD. Loeffler’s BacillusE. Bordet-Gengou bacillus

62. A 68-year-old patient had a stroke,after which she now often experiencesurges to vomit causing unilateral TMJ di-slocations. The dislocations could only bereduced under anesthesia. The 21-28, 15,16, 17, 18, 36, 37, 44, 46, 47 teeth are missi-ng. What denture construction is optimalfor this patient for the treatment period?

A. Removable lamellar denture with amouth opening stopperB. Schroeder appliance with sliding pivotC. Petrosov’s appliance with a stopperD. Yadrova’s applianceE. Khodorovich-Burgonskaya appliancewith a stopper

63. Parents of a 12-year-old child complainabout white patches on the upper frontteeth, which appeared six months ago.Objectively: chalk-like patches on thevestibular surfaces in the precervical areaof the 13, 12, 11, 21, 22, 23 teeth. Theirenamel is dull, pliable and rough on probi-ng. There is a history of short-term paincaused by chemical stimuli. What is yourprovisional diagnosis?

A. Acute initial cariesB. Chronic initial cariesC. Acute superficial cariesD. Systemic enamel hypoplasiaE. Dental fluorosis

64. A 3-year-old girl has been diagnosedwith acute odontogenous periostitis ofmandible starting from the 74 tooth. Itis required to perform periostomy andextract the 74 tooth. The child is excited.Choose the best type of anesthesia for thesurgery:

A. Intravenous anaesthesiaB. Mandibular anaesthesiaC. Infiltration anaesthesiaD. Intubation anaesthesiaE. Central anaesthesia

65. A 48-year-old patient complains ofpain in the 32, 38 teeth which arose3 months after installing a dental bri-dge upon them. Objectively: the gumsaround the abutment teeth are hyperemic,edematous, mobile in the vestibular-oraldirection. What is the most likely cause ofthis complication?

A. Functional overload of abutment teethparadontiumB. Improper preparation of abutmentteethC. The form of the intermediate denturepartD. Contact of the denture with opposingteethE. Solid food consumption

66. An 11-year-old child complains of acarious cavity in the left upper molar, whi-ch appeared six months ago. Objectively:the medial contact surface of the 26 toothexhibits a carious cavity located within the

Page 10: Krok 2 - 2011 Question Paper (Stomatology)

Kok 3 Stomatology 2011 10

mantle dentin. The cavity floor and wallsare dense, pigmented, with no pain uponprobing. The response to thermal stimuliis absent. Percussion of the tooth causesno pain. Make a provisional diagnosis:

A. Chronic median cariesB. Chronic deep cariesC. Acute median cariesD. Acute deep cariesE. Chronic periodontitis

67. A 42-year-old patient was diagnosedwith exacerbation of generalized gradeII periodontitis accompanied by abscessformation. What method of generaltreatment should be administered in thefirst place?

A. Antibiotic therapyB. Vitamin therapyC. Hyposensitization therapyD. Stimulating therapyE. Detoxification therapy

68. A 43-year-old patient complains ofincreased tooth sensitivity to thermal andchemical stimuli, gum itch. Objectively:the roots of the teeth are exposed to1/3 of their length, the gums are dense,pale pink. Thre is a small amount ofdental plaque. In the region of the 15,14, 24, 25, 44, 34 teeth there are wedge-shaped defects within the dentin. Probi-ng of exposed cervices and wedge-shapeddefects causes pain reaction. What is themost likely diagnosis?

A. ParodontosisB. Atrophic gingivitisC. Generalized periodontitisD. Gingival recessionE. Localized periodontitis

69. A 32-year-old patient complains aboutmouth soreness, body temperature riseup to 38, 5oC, indisposition. Such condi-tion has been occurring periodically forseveral years after the patient had had acold. Objectively: lips are covered withhaemorrhagic crusts, hyperaemic mucousmembrane of lips and cheeks has erosionscovered with fibrinous films. Hypersali-vation is present. What is the most likelydiagnosis?

A. Multiform exudative erythemaB. Pemphigus vulgarisC. Herpes recidivicusD. Herpetiform Duhring’s dermatitisE. Stevens-Johnson syndrome

70. Examination of a 5-year-old child

revealed a cariuos cavity communicati-ng with the tooth cavity on the approxi-mal surface of the 54 tooth. Halitosis ispresent. Superficial probing of the cavi-ty is painless, deeper probing causes painreaction. Percussion is painless. Intervi-ewing revealed that hot food causedpain. Select the medication for antiseptictreatment of root canals:

A. Sodium hypochloriteB. Aethonium solutionC. Furacilin solutionD. AlcoholE. Formalin solution

71. A 14-year-old teen complains ofgingival haemorrhages during toothbrushing. Objectively: gingival mucosa ishyperemic, pastous, bleeds when touched.Schiller-Pisarev test is positive. PMAindex - 70%. Hygienic index - 3,0. X-raypicture of the frontal area depicts no evi-dent changes. What is the most likely di-agnosis?

A. Chronic catarrhal gingivitisB. Chronic periodontitisC. Acute catarrhal gingivitisD. Chronic hypertrophic gingivitisE. Exacerbation of chronic periodontitis

72. A 50-year-old patient needs a splint forthe lower front teeth. Objectively: the 33,32, 31, 41, 42, 43 teeth present I-II classmobility and are devitalized. Which spli-nt would be the most appropriate in thiscase?

A. Mamlok-type splintB. Elbrecht’s splintC. Kurliandsky splintD. Murray’s splintE. Van Thiel splint

73. A 34-year-old male patient complai-ns about acute spasmodic pain in theregion of his upper jaw on the left thatis getting worse as affected by cold sti-muli. Toothache irradiates to the ear andtemple. He had acute toothache of the 37tooth one year ago, but he didn’t consulta dentist. Pain recurred three days ago.Objectively: the 37 tooth has a cariouscavity communicating with the dentalcavity. Probing of the opened cariouscavity is extremely painful. X-ray pictureshows widening of periodontal fissure atthe root apex of the 37 tooth. What is themost likely diagnosis?

Page 11: Krok 2 - 2011 Question Paper (Stomatology)

Kok 3 Stomatology 2011 11

A. Exacerbation of chronic pulpitisB. Exacerbation of chronic granulatingperiodontitisC. Exacerbation of chronic fibrous peri-odontitisD. Acute diffuse pulpitisE. Acute purulent pulpitis

74. A 30-year-old patient complains of apainless tumour-like formation in the regi-on of the left mandibular alveolar process.Objectively: there is a pale pink 1x1,5cm large tumour-like formation on thevestibular surface of the alveolar processat the 34 and 35 teeth. The formati-on has a broad base, it does not bleed.Crowns of the 34, 35 teeth are intact, theteeth are non-mobile. X-ray picture ofthe alveolar process shows no changes inbone structure. What is the most likely di-agnosis?

A. Fibrous epulisB. Pleomorphic adenomaC. Mucosa papillomaD. Giant-cell epulisE. Hypertrophic epulis

75. A 57-year-old patient complains ofmobility of all mandibular teeth. Objecti-vely: the 34, 35, 36, 45, 46, 47 teeth are mi-ssing. Orthopantomogram of the remaini-ng teeth shows resorption of the alveolarprocess within 1/3 of interdental septa hei-ght. What construction should be offeredto the patient?

A. Clasp denture with splinting appliancesB. Bridge dentures in the region of masti-catory teethC. Partial removable lamellar dentureD. Cap splintE. Van-Thiel splint

76. A 48-year-old patient consulted adental surgeon about extraction of the 37tooth’s roots. What tool should be chosen?

A. Closed-beak forcepsB. Broad-beak forcepsC. S-shaped forcepsD. Bayonet forcepsE. Curved beak-shaped forceps

77. A 35-year-old patient was diagnosedwith chronic median caries of the 36 tooth.There is a Black’s class II cavity affectingmasticatory surface. What material shouldbe chosen for the tooth filling?

A. Light-cure microhybride compositeB. Glass ionomer cementC. Silicophosphate cementD. Light-cure fluid compositeE. Light-cure microfilled composite

78. After the extraction of the 26 tootha 43-year-old patient presents with acommunication between the oral cavi-ty and the maxillary sinus. X-ray pictureshows no changes in the maxillary sinuses.What tactics should be chosen by a dentalsurgeon to prevent maxillary sinusitis?

A. Plastic restoring of the communicationB. Socket tamponade with a iodoformturundaC. Socket filling with a hemostatic spongeD. Sinus rinsing with an antibiotic solutionE. Socket filling with a blood clot

79. A 25-year-old patient complains ofintermittent pain during taking sweetand cold food in the 21 tooth. Objecti-vely: there is a hard tissue defect withinthe enamel in the cervical region of the21 tooth. The enamel edges are unevenand chalky. Probing the cavity bottomprovokes no pain, there is short-term painreaction to cold stimuli. What is the mostlikely diagnosis?

A. Superficial cariesB. Wedge-shaped defectC. Initial cariesD. Enamel hypoplasiaE. Enamel erosion

80. External examination of a 9-year-oldboy revealed strongly expressed nasolabi-al and labio-mental folds, a shortening ofthe lower third of face. Examination of theoral cavity revealed late transitional denti-tion, the upper front teeth completelyoverbite the lower teeth, the palate exhi-bits imprints of the lower incisors. What isthe most likely diagnosis?

A. SupraocclusionB. Dentoalveolar maxillary lengtheningC. Distal occlusionD. Dentoalveolar mandibular lengtheningE. Mesial occlusion

81. A 70-year-old patient complains aboutthe mobility of the 21 tooth. The toothhas not been treated before. Objectively:there is II class mobility of the 21 tooth,the root is exposed by 1/2. What toolshould be used to extract the tooth?

Page 12: Krok 2 - 2011 Question Paper (Stomatology)

Kok 3 Stomatology 2011 12

A. Straight forcepsB. S-shaped forcepsC. Bayonet forcepsD. Straight elevatorE. Beak-shaped forceps

82. Following the upper jaw examinati-on (according to Schroeder) a 65-year-old patient was found to have atrophi-ed maxillary tubers, alveolar processescovered with atrophic, thin and palemucous membrane. What kind of functi-onal impression technique should beapplied?

A. DecompressiveB. CompressiveC. DosatedD. Pressure-freeE. Differentiated

83. An orthodontist monitors a 4-year-oldchild with mouth breath. The child hasa history of adenotomy. Objectively: pri-mary dentition occlusion; the upper inci-sors overlap the lower ones by 1/3; distalsurfaces of the second temporary molarsare situated in the same vertical plane.What preventive device will help the childto give up the habit of mouth breath?

A. Standard Schonherr’s vestibular screenB. Vesibular and oral Kraus’ screenC. Frankel’s function regulatorD. Andresen-Haupl activatorE. Rudolph’s appliance

84. A 17-year-old patient consulted anorthodontist about improper position ofan upper canine. Objectively: permanentocclusion, class I Angle’s relationship ofthe first molars, the 13 tooth has vestibularposition above the occlusal line, there is a6,5 mm gap between the 14 and 12 teeth.What period of orthodontic treatment wi-ll reduce the time of lidase phonophoresistherapy?

A. Active periodB. Retention periodC. Preparatory periodD. Passive periodE. -

85. Preventive examination of a 55-year-old man revealed a well-defined area ofopaque mucosal epithelium of the leftcheek that didn’t protrude above thesurrounding tissues and could not beremoved on scraping. Crowns of the 34,35, 36 teeth were strongly decayed andhad sharp edges. What is the most likelydiagnosis?

A. LeukoplakiaB. Lichen ruber planusC. Lupus erythematosusD. CandidiasisE. Soft leukoplakia

86. A 62-year-old patient presented tothe prosthodontics clinic with complai-nts about poor fixation of completeremovable lamellar dentures that showsup during talking and wide mouth openi-ng. The dentures were fabricated 5 daysago. In course of denture fabricationprocess a mistake was made during:

A. Obtaining the functional impressionB. Occlusal plane orientationC. Interalveolar dimension measurementD. Wax reproduction checkE. Plastic polymerization

87. A 30-year-old patient has a cariouscavity in the 11 tooth within parapulpardentin. The tooth corner and its cuttingedge are affected. What filling material isbest for restoration of the anatomic toothform?

A. Light-cure compositeB. Silicate cementC. Light-cure glass-ionomer cementD. Chemical-cure paste-paste compositeE. Chemical-cure powder-liquid composite

88. A 29-year-old patient complains ofacute paroxysmal pain in the upper jawon the left, that gets worse during havi-ng cold food and irradiates into the earand temple. A year ago she experiencedintense pain in the 27 the tooth but didn’tconsult a dentist. Three days ago therewas the second attack of pain. Objecti-vely: there is a deep carious cavity in the27th tooth, interconnecting with the toothcavity. Probing the open area causes acutepain. What is the most likely diagnosis?

A. Exacerbation of chronic pulpitisB. Acute serous periodontitisC. Acute diffuse pulpitisD. Exacerbation of chronic periodontitisE. Acute limited pulpitis

89. A 23-year-old patient complains ofwhitish spots on the masticatory teeth,drawing of mouth during taking acidicfood. The spots appeared about 3 monthsago. Objectively: the cervical region ofthe 46, 36, 27 teeth exhibits some chalk-like spots that can be easily stained with2% methylene blue, probing reveals thesurface roughness. What is the most likely

Page 13: Krok 2 - 2011 Question Paper (Stomatology)

Kok 3 Stomatology 2011 13

diagnosis?

A. Acute initial cariesB. Acute superficial cariesC. Enamel hypoplasiaD. Endemic fluorosisE. Chronic superficial caries

90. A 56-year-old patient has an oval,smooth, bright-red erosion on the redborder of her lower lip. Erosion is coveredwith haemorrhagic crusts that can behardly removed. Crust removal inducesslight haemorrhage. Light traumatizationof crust-free surface of erosion induces nohaemorrhage. Specify the type of lowerlip precancer:

A. Abrasive precancerous Manganotti’scheilitisB. Verrucous precancer of red borderC. Localized precancerous hyperkeratosisof red borderD. Bowen’s diseaseE. Erythroplasia

91. A 55-year-old patient consulted adentist about a roundish tumour-likeformation of about 1 cm in diameterlocated within the red border of his lowerlip. Objectively: the tumor-like formati-on protrudes about 5 mm above the redborder, it is dense and grayish-red. Thesurface of the formation is covered withthin scales that can be hardly removed.What is the most likely diagnosis?

A. Verrucous precancer of the red borderof lipB. Abrasive precancerous Manganotti’scheilitisC. Precancerous limited hyperkeratosis ofthe red border of lipD. Bowen’s diseaseE. Erythroplasia of Queyrat

92. A 4,5-year-old child presents witheruptions on skin and in the mouth whichappeared on the previous day. Objecti-vely: the child is in medium severe condi-tion, body temperature is 38, 3oC. Scalp,trunk skin and extremities are coveredwith multiple vesicles with transparentcontent. Mucous membrane of cheeks,tongue, hard and soft palate exhibitsroundish erosion covered with fibrinousfilm. Gums remain unchanged. Submandi-bular lymph nodes are slightly enlarged.What diagnosis can be assumed?

A. Chicken pox-induced stomatitisB. Acute herpetic stomatitisC. Exudative erythema multiformeD. Measles-induced stomatitisE. Scarlet fever-induced stomatitis

93. A 47-year-old patient with completelyedentulous upper jaw underwent delayedimplantation with cylindrical implants.Prosthetics is recommended in:

A. 5-6 monthsB. 24 hoursC. 2 weeksD. 1 monthE. 3 months

94. Preventive examination of a 5-year-oldchild reveals the infantile swallowing. Thebad habit of thrusting the tongue betweenthe teeth may cause the following compli-cation:

A. Incomplete eruption of the front teethB. Broadenning of the upper dental archC. Broadening of the lower dental archD. Narrowing of the lower dental archE. Narrowing of the upper dental arch

95. After a preventive orthodontic exami-nation a 9-year-old child was diagnosedwith mesial occlusion. The treatment ofthis pathology involves application of anapparatus with mechanic action. Whatworking element is to be applied in theapparatus intended for the correction ofthis pathology?

A. Screw or springB. Occlusal applicationsC. Elastics and buccal shieldsD. Inclined planeE. Screw and bite plate

96. A 28-year-old male patient presentedto a clinic on the 2nd day after theface trauma and complained of bilateralmandibular pain, difficult swallowing andobstructed breathing. Objectively: therewas a slight swelling in the region of themolars on both sides, the teeth were intactbut the jaw fragment with the 43, 42,41, 31, 32, 33 teeth deviated downwardand backward. Manual reposition didn’tproduce any positive result. Which appli-ance should be used for treatment?

A. Post’s applianceB. One-jaw Katz applianceC. One-jaw Kurliandsky applianceD. Brun applianceE. Betelman appliance

Page 14: Krok 2 - 2011 Question Paper (Stomatology)

Kok 3 Stomatology 2011 14

97. An 18-year-old patient presented tothe orthopedic department with complai-nt of a large diastema. Objectively: bodi-ly lateral movement of central incisorsinduced by the missing 12, 22 teeth. Whatis the optimal appliance for moving thecentral incisors together?

A. Korkhaus apparatusB. Vasilenko apparatusC. Cotton ligatureD. Kalvelis apparatusE. Begg appliance

98. A 30-year-old patient needs to have his26 tooth extracted because of exacerbati-on of chronic periodontitis. Objectively:the crown of the 26 tooth is decayed by1/3. What forceps can be used for thistooth extraction?

A. S-shaped forceps with a projecting tipon the left beakB. S-shaped forceps with a projecting tipon the right beakC. Straight forcepsD. Straight elevatorE. S-shaped forceps without projecting tips

99. A 26-year-old patient consulted adentist about the 24 tooth extracti-on because of chronic periodontitisexacerbation. The crown of the 24 tooth isintact. Choose the right tool for the toothexraction:

A. S-shaped forceps without projectingtipsB. S-shaped forceps with a projecting tipon the left beakC. S-shaped forceps with a projecting tipon the right beakD. Straight forcepsE. Straight elevator

100. A 49-year-old patient consulted adental surgeon about the oral cavitysanation. He has an indication for theextraction of the 16 tooth. Anamnesis: thetooth hasn’t been treated before, decayedwithin the last 4 years. Objectively: the16 tooth’s crown is destroyed by over2/3, the mucosa exhibits no pathologicalchanges. Which tool is required for thetooth extraction?

A. Bayonet root forcepsB. S-shaped forceps (right)C. Straight elevatorD. S-shaped closed-beak forcepsE. Crown bayonet forceps

101. Mother of a 3-year-old child complai-

ns about white spots on the upper anteriorteeth of her child. Objectively: the vesti-bular surface of the 51 and 61 teeth exhi-bits a defect within the enamel. Probingis painless. What is the optimal method oftreatment?

A. Impregnation with silver nitrate soluti-onB. Treatment with fluorine lacquerC. Remodentum applicationD. Preparation and fillingE. Electrophoresis of sodium fluoridesolution

102. A 20-year-old patient is to be givenapplicative anaesthesia in the frontal regi-on of the upper jaw prior to the conducti-on anesthesia. What concentration of li-docaine solution is required for applicati-ve anesthesia?

A. 10%B. 0,5%C. 2%D. 3%E. 15%

103. A 13-year-old boy complains of painin the region of the extracted 46 tooth,irradiating to the ear and temple, hali-tosis. The tooth was extracted 3 daysago. Objectively: submandibular lymphnodes are enlarged, painful on palpati-on. Mucosa around the extracted tooth ishyperemic, edematous. The socket of the46 tooth is filled with a gray clot. What isthe most likely diagnosis?

A. Alveolitis of the extracted 46 toothB. Acute odontogenous mandibularosteomyelitis starting from the 46 toothC. Acute odontogenous lymphadenitis ofthe right submandibular regionD. Acute odontogenous mandibular peri-ostitis starting from the 46 toothE. Neuralgia of the III branch of trigemi-nus

104. A 35-year-old patient complainsabout progressing throbbing pain in the26 tooth. Objectively: the 26 tooth has acarious cavity filled with softened denti-ne, tooth cavity is closed, probing of thecavity floor is painless, percussion causesacute pain. There is I degree tooth mobi-lity. Roentgenological changes are absent.What is the most likely diagnosis?

Page 15: Krok 2 - 2011 Question Paper (Stomatology)

Kok 3 Stomatology 2011 15

A. Acute purulent periodontitisB. Acute purulent pulpitisC. Acute serous periodontitisD. Exacerbation of chronic periodontitisE. Acute diffuse pulpitis

105. A 43-year-old patient consulted adentist about pain in the 36th tooth. X-ray examination revealed a breakage ofan endodontic tool in the distal root. Theroot apex exhibits a well-defined area ofbone tissue destruction (d = 5 mm) withclear margins. What method of treatmentis most appropriate in this case?

A. Dental hemisectionB. Root apex resectionC. Conservative treatmentD. Tooth replantationE. Root amputation

106. A 25-year-old patient has a facetrauma. Objectively: there is a signi-ficant soft tissue swelling of the lefthalf face, haemorrhage into the lefteye sclera, crepitation in the region ofnose bones. Palpation reveals the mobi-lity of the upper jaw, its percussionreveals tympanitis. X-ray picture showsthe fracture line running through thelower margin of the orbit on both sides ofthe sphenozygomatic suture and reachingbehind the maxillary tuber. What is themost likely diagnosis?

A. Le Fort II maxillary fractureB. Le Fort I maxillary fractureC. Le Fort III maxillary fractureD. Fracture of the left articular boneE. Fracture of nose bones

107. A 57-year-old patient complains of atumor in the left parotid region, whichappeared 3 years ago. Over the past sixmonths, the tumor has increased signifi-cantly. Objectively: there is a non-mobilelobulated tumour in the left parotid regi-on, palpation causes minor pain reaction.The skin over the tumour plicates easily,its upper part has well-defined margins,the bottom part grows into the depth ofgland. The tumour is 3x2,5 cm large. Whatis your provisional diagnosis?

A. Pleomorphic adenomaB. Chronic parotitisC. FibromaD. CystE. Parotid gland carcinoma

108. A 53-year-old patient complains of atumour in the right parotid region. Thetumour was revealed six months ago, wi-

thin this period it doubled in size. Objecti-vely: the right parotid region has a dense1,5 x 2 cm large neoplasm, the duct of theright parotid gland secretes transparentsaliva. The gland puncture gave no results.What disease are these clinical presentati-ons most typical for?

A. Pleomorphic adenoma of parotid glandB. Chronic lymphadenitis of parotid-masticatory regionC. Retention cyst of parotid glandD. Fibroma of parotid-masticatory regionE. Lipoma of parotid-masticatory region

109. A 41-year-old patient complainsof mobility of the 24, 26, 27 teeth,purulent discharges from the socket ofthe extracted 25 tooth. 1,5 months agothe patient underwent a dissection alongthe mucogingival junction and extracti-on of the 25 tooth. Objectively: alveolarmucosa in the region of the 24, 26, 27teeth is cyanotic and edematic. Along themucogingival junction there is a fistulawith protruding granulations. There arealso purulent granulation discharges fromthe socket of the extracted 25 tooth. Inthe right infraorbital region some soft ti-ssue swelling is present. Which disease arethese clinical presentations most typicalfor?

A. Chronic limited osteomyelitisB. Chronic alveolitisC. Maxillary actinomycosisD. Exacermation of chronic maxillarysinusitisE. Chronic diffuse osteomyelitis

110. A 37-year-old patient complai-ns of gingival haemorrhage, offensivebreath, tooth mobility, difficult masticati-on. Objectively: the gums are of cyanotic-red colour, periodontal pockets are 6mm deep and contain serous exudate,there is II class tooth mobility, moderateamount of subgingival and supragingivaltartar; hygiene index is 3 points, thereis traumatic occlusion along the entirelength of the dental arches. What is yourprovisional diagnosis?

A. Chronic generalized grade II peri-odontitisB. Acute generalized grade I periodontitisC. Hypertrophic gingivitisD. Histiocytosis-XE. Acute generalized II grade periodontitis

111. Parents of a 12-year-old childconsulted an orthodontist about improperposition of the child’s upper teeth.

Page 16: Krok 2 - 2011 Question Paper (Stomatology)

Kok 3 Stomatology 2011 16

Objectively: the face is narrow, elongated;the developing occlusion is present(temporary second molars). The 13 and 23teeth are located beyond the dental arch,they deviate to the lips above the occlusalplane, there is a 2,5 mm gap between the12 and 14 teeth, and a 1,5 mm gap betweenthe 22 and 24 ones, 45o rotation the 33 and43 teeth is present. Choose the most rati-onal method of treatment:

A. Extraction of the premolars andrelocation of the caninesB. Expansion of dental arches in the regionof canine apicesC. Extraction of temporary premolars andexpansion of dental archesD. Compact osteotomy and expansion ofdental archesE. All the answers are wrong

112. Before extraction of a left uppertooth a 49-year-old patient had been givenplexus anaesthesia with Ultracaine Forte.After the operation the socket didn’t getfilled with a blod clot. What is the wayto prevent alveolitis development in thispatient?

A. To fill the socket with a loose iodoformtamponB. To fill the socket with a hemostaticspongeC. To rinse the socket with microcidesolutionD. To rinse the socket with 0,1% chlorhexi-dine solutionE. To fill the socket with antibiotic powder

113. Examination of a 45-year-old pati-ent revealed some white patches on thebuccal mucosa along the teeth closure li-ne that didn’t protrude above the inflamedand edematous surrounding tissues. Thepatches could not be removed on scrapi-ng. A patient has smoken an average of3 packs of cigarettes a day for 20 years.Specify the disease of buccal mucosa:

A. Leukoplakia planaB. Erosive leukoplakiaC. PemphigusD. Tappeiner’s leukoplakia (leukoplakianicotinica)E. Lichen ruber planus

114. X-ray examination of the maxillaryalveolar process of a 35-year-old patientreveals a root fracture of the 11 tooth in itsapical region. The tooth has been treatedbefore, the canal is filled. The patient has ahistory of the tooth trauma. What method

of treatment is indicated for this case?

A. Root apex resectionB. Tooth extractionC. Tooth crowningD. Tooth replantationE. -

115. During the examination of a soldereddental bridge supported by the 35 and 38teeth the following flaws were revealed:the pores in the soldered joints betweenthe intermediate part and supportingcrowns, strongly expressed masticatorytubercles, early contact with the opposi-ng teeth, the intermediate part is tightlyadjacent to the mucous membrane of thealveolar process. What is the way to elimi-nate these defects?

A. To remake the dental bridgeB. To correct the intermediate partC. To correct the masticatory surface andpolish the jointsD. To correct the masticatory surface andincrease the height of the intermediatesaddle part up to 2 mmE. To remove the tubercles of opposingteeth

116. A 42-year-old woman came toprosthetic dentistry to make dentalprosthetics. Objectively: dental formula is

18 . . . . 13 12 11 21 22 23 . . . . 2848 47 46 45 44 43 42 41 31 32 33 34 35 36 37 .

The bite is deep, clinical crowns are low,survey line is not marked. The patientsuffers from epileptiform attacks. Whatprosthesis is indicated?

A. Partial removable lamellar prosthesiswith metal baseB. Dental bridgesC. Partial removable lamellar plasticprosthesis with retaining claspsD. Partial removable lamelalr prosthesiswith supporting-retaining claspsE. Clasp denture

117. A 43-year-old female patientcomplains of mobility of the lower frontteeth. Objectively: the 48, 47, 46, 45, 44,34, 35, 36, 37, 38 teeth are missing. The43, 42, 41, 31, 32, 33 teeth exhibit gradeII mobility, the mucous membrane aroundthem is swollen and cyanotic. What spli-nting construction is optimal for this pati-ent?

Page 17: Krok 2 - 2011 Question Paper (Stomatology)

Kok 3 Stomatology 2011 17

A. Composite splint reinforced with fi-berglassB. Partial removable denture with a vesti-bular barC. Clasp denture with multilink clasp andclaw-like processesD. Clasp denture with antithrowersE. Clasp denture-splint in the frontal part

118. A 58-year-old patient was diagnosedwith fracture of lower jaw with formati-on of a false joint. Objectively: the 38, 36,32, 41, 43, 48 teeth are missing. Preservedteeth are intact, stable. There is no di-splacement of lower jaw fragments. X-raypicture shows a bone tissue defect to up 2cm large. What construction of prosthesisis indicated in this situation?

A. Fixed Oxman’s prosthesis with pivotpointB. Oxman’s single-jointed prosthesis withpivot pointC. Gavrilow’s prosthesis with pivot pointD. Kurlyandsky prosthesis with rollerdamper claspE. Oxman’s double-jointed prosthesis withpivot point

119. A 15-year-old girl consulted a dentalsurgeon about a moderate swelling of theparotid-masticatory region on the left.Objectively: on palpation, the left parotidgland is nodular, dense, painless. The ductexudes somewhat turbid saliva. The ductmouth is dilated, the surrounding mucousmembrane is cyanotic, pasty, with imprintsof teeth. Make a diagnosis:

A. Chronic parenchymatous sialoadenitisB. Chronic interstitial sialoadenitisC. SialolithiasisD. Pleomorphic adenomaE. Cyst

120. A 53-year-old patient presentedto a dental clinic for the orthopaedictreatment of the 27 tooth. Duringpreparation for local anesthesia, thepatient felt sick (weakness, pallor,hyperperspiration), and lost consci-ousness. What is the most likely di-agnosis?

A. SyncopeB. CollapseC. Anaphylactic shockD. Traumatic shockE. Cardiogenic shock

121. A 27-year-old patient complains ofbleeding, pain and swelling of gums, toothmobility, halitosis, general weakness.

Objectively: submandibular lymph nodesare enlarged and painful, gums arehyperemic and cyanotic, they are alsoswollen and bleed at probing, periodontalpockets are 5-8 mm deep and containsero-purulent exudate. There is massi-ve dental deposit, gingival recession andtooth migration. X-ray picture shows di-ffuse osteoporosis and irregular, predomi-nantly vertical, resorption of alveolarsepta down to 2/3 of root length. Whatis the most likely diagnosis?

A. Exacerbation of generalized grade IIIperiodontitisB. Grade III parodontosisC. Eosinophilic granulomaD. Chronic generalized grade III peri-odontitisE. Exacerbation of generalized grade IIperiodontitis

122. A 37-year-old patient complains of gi-ngival haemorrhage, pain, halitosis, bodytemperature rise up to 37, 6oC. Objecti-vely: the gums are markedly hyperemic,swollen, bleed on touch, periodontalpockets are 3 mm deep, contain sero-purulent exudate. Orthopantomogramshows diffuse osteoporosis of alveolarprocesses, resorption of the interdentalsepta down to 1/3 of their height. What isthe most likely diagnosis?

A. Generalized grade I periodontitisB. Exacerbation of chronic catarrhal gingi-vitisC. Generalized grade II periodontitis,chronic courseD. Generalized grade II periodontitis,exacerbationE. Generalized grade I periodontitis,chronic course

123. A 21-year-old patient complains of adrop-shaped bulge in the cervical region.Objectively: there is an enamel droplet ofabout 2 mm in diameter on the vestibularsurface of the 33 tooth. What is the mostlikely diagnosis?

A. Enamel hyperplasiaB. Enamel hypoplasiaC. FluorosisD. Supernumerary toothE. Marble-bone disease

124. A 49-year-old male patient complainsabout gingival haemorrhage, teeth mobi-lity in the frontal region of his lower jaw,hypersensitivity of dental cervixes. X-raypicture shows widening of periodontal fi-ssure in the region of the 42, 41, 31, 32

Page 18: Krok 2 - 2011 Question Paper (Stomatology)

Kok 3 Stomatology 2011 18

teeth, as well as resorption of alveolarprocess tissue to 1/3 of root length. The42, 32 teeth present with I degree mobili-ty, the 41, 31 teeth present with II degreeof teeth mobility. What temporary splintshould be used for frontal stabilization ofdentition?

A. NovotnyB. MamlockC. TreumanD. KoganE. Kopeinin

125. A 7-year-old child presents withdeterioration of general health status,body temperature rise up to 38, 0oC.Objectively: buccal mucosa is coveredwith white maculae of a pinhead size,protruding above the level of the mucosa.There are pink maculae on the palate.What is the most likely diagnosis?

A. MeaslesB. Chicken poxC. Scarlet feverD. DiphtheraE. Infectious mononucleosis

126. A 32-year-old patient needs tobe provided with metal-ceramic crownsfor the 12, 11, 21 and 22 teeth. Duri-ng the dental visit he is given infi-ltration anaesthesia with Ultracain DSanaesthetic. What elements enter into itscomposition?

A. 4% articaine with adrenalineB. 2% mepivacaine with adrenalineC. 4% articaine without a vasoconstrictoragentD. 3% mepivacaine without a vasoconstri-ctor agentE. 2% articaine with epinephrine

127. A 25-year-old got a polytrauma asa result of a road accident. On exami-nation, he was diagnosed with dislocationasphyxia. What kind of first aid should berendered?

A. Tongue fixationB. TracheotomyC. Artificial ventilation of lungsD. Medicamental management oflaryngeal stenosisE. Removal of mucus and foreign bodiesfrom the upper airways

128. A 7-year-old child has been di-agnosed with a complete dislocation ofthe 11 tooth caused by a trauma thatoccurred 24 hours ago. The tooth has been

taken along with the patient to the denti-st’s. Objectively: the alveolar socket edgesare slightly hyperemic, the socket bonewalls are intact, the socket itself is filledwith a blood clot. What is the optimal wayof treatment?

A. Replantation of the 11th tooth with aprior filling of the channelB. Replantation of the 11th tooth beforethe filling of the channelC. Extraction of the 11th toothD. Reposition and fixation of the 11thtoothE. Socket suturing with catgut

129. A 20-year-old patient got an injury.Objectively: the patient’s chin and lowerjaw up to the 34 and 45 teeth are missing.The 45, 46, 47, 48, 34, 35, 36, 37 teeth arestable. At what stage of medical evacuati-on the patient will get special medical aid?

A. Specialized army surgical hospitalB. Battalion aid stationC. Regimental aid stationD. Separate medical detachmentE. Separate medical battalion

130. A 25-year-old patient complainsabout a light brown spot in the upperforetooth. Objectively: the 23 tooth has asingle light brown spot in the cervical regi-on. Probing shows smooth surface. Thetooth is nonresponsive to cold and probi-ng. What is the most likely diagnosis?

A. Chronic initial cariesB. FluorosisC. Local enamel hypoplasiaD. Acute initial cariesE. Chronic superficial caries

131. A 33-year-old patient complai-ns about an ulcer of oral cavity floor,that is located under his tongue on alevel between the 43 to the 33 tooth.Examination reveales that ulcer edgesare undermined and scalloped. Its grey-yellow floor is shallow and it is coveredwith small, easily bleeding granulations.There is no ulcer infiltration. Make a cli-nical diagnosis:

A. Tubercular ulcer of oral cavity floorB. Decubital ulcer of oral cavity floorC. Cancerous ulcer of oral cavity floorD. Gummatous ulcer of oral cavity floorE. Migratory granuloma of oral cavity floor

132. A 60-year-old patient presentedto a clinic for checking her completeremovable denture. During the teeth

Page 19: Krok 2 - 2011 Question Paper (Stomatology)

Kok 3 Stomatology 2011 19

placement on the wax bases a gapbetween the front teeth and cuspalinterference of the lateral teeth wasrevealed. What mistake was made?

A. The patient was found to have anteriorocclusion instead of centric oneB. The patient was found to have posteriorocclusion instead of centric oneC. The patient was found to have lateralocclusion instead of centric oneD. The models were improperly plasteredin the occludatorE. During establishing the centric occlusi-on, the occlusal rims were overpressed

133. A 45-year-old female patientconsulted a prosthodontist about dentalprosthetics. She works as TV announcer.Objectively: the lower jaw dentition is wi-thout spaces, the upper jaw has a free-end edentulous space and a boundaryedentulous space in the lateral parts. Theremaining 18, 17, 13, 12, 11, 21, 22, 23,24 teeth are stable. What fixing elementsshould be used in a clasp denture forcosmetic purposes?

A. Attachments and bar systemB. Telescopic crownsC. Ney claspsD. Jackson claspsE. Dentoalveolar clasps

134. A 53-year-old patient with generali-zed periodontitis presented to aprosthodontics clinic for fabrication offixed splints for his both jaws intendedto stabilize the dentitions along the enti-re dental arch. Which of the upper jawbuttresses will be functionally signifi-cant for the distribution of masticatorypressure?

A. Frontonasal, zygomatic, pterygopalati-ne, palatineB. Frontonasal, zygomatic, pterygopalatineC. Zygomatic, pterygopalatine, palatineD. Frontonasal, pterygopalatine, palatineE. Frontonasal, zygomatic, palatine

135. A 38-year-old patient complains ofdiscomfort, occasional sensation of teari-ng in the 17 tooth, infrequent pain duringeating hot food that lasts for 10-20 mi-nutes. Objectively: the 17 tooth is filledwith composite materials, the tooth is sli-ghtly darker than the other ones. What isthe most likely diagnosis?

A. Chronic gangrenous pulpitisB. Chronic granulating pulpitisC. Chronic concrementous pulpitisD. Recurrent cariesE. Chronic fibrous pulpitis

136. On the 7th day after the operation onthe abdominal cavity a 30-year-old pati-ent presented with pain and sweling in theregion of parotid gland; body temperaturerise up to 39oC; limited mouth opening,dryness. Gland massaging results in di-scharge of purulent exudate from its duct.The patient can be diagnosed with:

A. Acute non-epidemic parotiditisB. Acute epidemic parotiditisC. Phlegmon of submasseteric spaceD. Parenchymatous parotiditisE. Phlegmon of parotidomasseteric region

137. A 50-year-old patient complains ofa traumatic break-off in the front teeth.Objectively: the crowns of the 12, 21 teethare broken off by 3/4, X-ray picture showsthat the channels of the 11, 21 teeth areproperly filled, the other teeth are intact.What actions should the dentist take?

A. Fabricate fullcast metal stumps with apost and restorative metal-ceramic crownsB. Remove the 11 and 21 teeth and fabri-cate a dental bridge supported by the 12and 22 teethC. Remove the 11 and 21 teeth and fabri-cate a partial removable lamellar denturefor the upper jawD. Fabricate restorative swaged metalcrowns for the 11 and 21 teethE. Fabricate a dentogingival plastic splint

138. A 16-year-old girl consulted a denti-st about a cosmetic defect on her frontteeth, namely some dark spots and dentindefects. The spots were detected immedi-ately after teeth eruption, the defectsdeveloped with the lapse of time. Thepatient was diagnosed with erosive formof dental fluorosis of the 16, 11, 12, 22, 26,31, 32, 36, 41, 42, 46 teeth. Select a methodof treatment:

A. RestorationB. RemineralizationC. ProstheticsD. Surgical treatmentE. Regular surveillance

139. During extration of the 26 tooth a 34-year-old patient developed tinnitus, skinpallor, weakness. She was diagnosed wi-th vertigo. What is the most appropriatetactics of emergency care?

Page 20: Krok 2 - 2011 Question Paper (Stomatology)

Kok 3 Stomatology 2011 20

A. Placing the patient in TrendelenburgpositionB. Intravenous injection of 50% analginC. Intravenous injection of 0,1% adrenalinD. Intravenous injection of cordiaminE. Intravenous injection of prednisolone

140. A 30-year-old male patient consulteda dentist about a swelling in the regionof his upper lip. Objectively: the face isasymmetric because of upper lip edema,nasolabial fold is smoothed. Mucogingivalfold of the upper jaw vestibular surfacein the region of the 11, 12, 21 teeth issmoothed, hyperemic. Palpation is pai-nful. Fluctuation is present. The patientwas diagnosed with acute purulent peri-ostitis of the upper jaw satrting from the21 tooth. Choose the treatment tactics:

A. Preservation of the 21 tooth, peri-osteotomy, anti-inflammatory therapyB. Preservation of the 21 tooth, anti-inflammatory therapyC. Extraction of the 21 tooth, peri-osteotomy, anti-inflammatory therapyD. Extraction of the 21 tooth, peri-osteotomyE. Extraction of the 12, 21, 22 teeth,periosteotomy, anti-inflammatory therapy

141. A 37-year-old patient was admitted tothe oral surgery department. The womancomplains of pain in her cheek that ismade worse by touch. It is known from theanamnesis that five days ago the patientfell down from the stairs and "hurt"hercheek. Objectively: in the depth of cheekexamination revealed a circumscribedinfiltration, cheek skin is hyperaemic anddoesn’t make a fold, fluctuation symptomis present. Mucous membrane is edematicand has teeth indentations. What is themost likely diagnosis?

A. Suppurated cheek haematomaB. Traumatic osteomyelitis of mandibleC. Cheek haematomaD. Cheek phlegmonE. Acute lymphadenitis

142. A 42-year-old builder complainsabout a condyloma on his lower lip. Itappeared 1,5 month ago. It has been si-gnificantly growing throughout the lastweek. Objectively: the red border of thelower lip is cyanotic and infiltrated, it hassome isolated closly adhering squamae.There is a well-defined hemisphericalformation 8 mm in diameter and 4 mmhigh in the centre. The formation is ofgrey-and-blue-and-red colour, it has rough

surface formed by thin, closely adheri-ng and thick-based squamae. Regionallymph nodes are enlarged, mobile, denseand painless. What is the most likely di-agnosis?

A. Lower lip cancerB. Verruciform precancerC. Pyogenic granulomaD. KeratoacanthomaE. Viral wart

143. A 12-year-old child presents with mi-ssing 31 and 41 teeth, the gap between the32 and 42 teeth is 10 mm. Choose the mostrational denture construction:

A. Partial lamellar removable adjustabledentureB. Interdental wedgeC. Clasp dentureD. Dental bridgeE. Adjustable microprosthesis

144. A 6,5-year-old child has a gap 2,5-3 mm large between frontal teeth fromcanine to canine. Relationship of the firstpermanent molars complies with Angle’sclass I. Specify the severity degree of bitedeformation:

A. I degreeB. II degreeC. III degreeD. IV degreeE. V degree

145. An 8-year-old child complains ofa painless round formation under thetongue on the left which was noticed byaccident about a month ago. Objectively:there is a roundish formation up to 2,0cm in diameter in the sublingual regiondeep in the soft tissues of mouth floor. Itis soft, painless to palpation, the mucousmembrane over it is of grayish-blue color.What is the most likely diagnosis?

A. Ranula of the left sublingual salivaryglandB. Dermoid cyst of the mouth floorC. Epidermoid cyst of the mouth floorD. Lymphangioma of the mouth floorE. Hemangioma of the mouth floor

146. X-ray examination of the 46 toothin a 7-year-old child reveals the followi-ng: the root walls are parallel, their thi-ckness gradually decreases, they havepointed ends. The root canal convergestowards the tooth cavity, and diverges atthe developing apical hole. Periodontalgap is of the same width along the enti-

Page 21: Krok 2 - 2011 Question Paper (Stomatology)

Kok 3 Stomatology 2011 21

re length of the root. At the root apex itmerges with the growth zone. Specify thestage of root development:

A. Continuing root apex developmentB. Continuing root developmentC. Open apexD. Continuing periodontium developmentE. Complete root and periodontiumdevelopment

147. A 5-year-old child complains aboutspontaneous pain in an upper jaw tooth onthe right that is getting worse at night andduring eating cold food. Objectively: the65 tooth has a deep cavity communicatingwith the tooth cavity. Probing is painful,percussion is painless. Cold water causeslong-standing pain. What is your provisi-onal diagnosis?

A. Exacerbation of chronic pulpitisB. Acute periodontitisC. Exacerbation of chronic periodontitisD. Acute serous pulpitisE. Acute purulent pulpitis

148. Parents of a 6-year-old child complainabout pain in the child’s submandibularregion on the left, body temperature ri-se up to 37, 5oC. Objectively: the child’sface is asymmetric due to the infiltrationof the submandibular region on the left.The infiltration is soft and elastic, mobile,2х2,5 cm large; its palpation is slightly pai-nful, the skin is unchanged. The teeth areintact. Pharynx is hyperaemic. What is themost likely diagnosis?

A. Acute serous nonodontogenicsubmandibular lymphadenitisB. Acute serous odontogenic submandi-bular lymphadenitisC. Acute purulent nonodontogenicsubmandibular lymphadenitisD. Acute purulent odontogenic submandi-bular lymphadenitisE. Submandibular adenophlegmon

149. A dentist performing gum anestheti-zation before the closed curettage had mi-stakenly used the 10% solution of silvernitrate instead of 10% lidocaine soluti-on. The gums immediately became white,swollen, painful. Which of these medicati-ons is to be used for emergency care?

A. 3% solution of potassium iodideB. 0,5% solution of acetic acidC. 0,1% solution of liquid ammoniaD. 2% solution of citric acidE. 50% of ethyl alcohol

150. A patient complains aboutparoxysmal upper jaw toothache onthe left that is getting worse at night.Toothache intensifies also under stimulati-on and irradiates to the left eye andtemple. Similar attacks were noted threemonths ago, the patient didn’t undergoany treatment. Objectively: the 25 toothhas a deep carious cavity communicati-ng with the tooth cavity. Probing causesacute pain at the point of communicati-on, vertical percussion is slightly pai-nful, horizontal one is painless. Mucousmembrane in the projection of root apexof the 25 tooth is unchanged, its palpati-on is painless. Thermal probe causesacute pain, the pain attack is long-lasting. Electroodontodiagnosis is 60 mi-croampere. X-ray picture shows slight wi-dening of periodontal fissure at the rootapex of the 25 tooth. What is the mostprobable diagnosis?

A. Exacerbation of chronic pulpitisB. Acute generalized pulpitisC. Acute purulent pulpitisD. Acute purulent periodontitisE. Exacerbation of chronic periodontitis

151. A 48-year-old woman complainsabout aching dull pain in the region ofthe left TJM, that is getting worse duri-ng eating solid food. The pain appearedabout 2,5 years ago. Objectively: mouthopening is limited, there is sideward devi-ation of jaw during mouth opening, TMJis clicking. Examination of the oral cavi-ty revealed secondary partial adentia. X-ray picture shows sclerosis of the corticalplate of articulat head and narrowing ofcartilage space. What is the most likely di-agnosis?

A. Arthrosis of the TMJB. Chronic arthritis of the TMJC. Acute arthritis of the TMJD. Painful dysfunction of the TMJE. Exacerbation of chronic arthritis of theTMJ

152. A patient with haemophilia consulteda dental surgeon about decay of a rightlower tooth. Objectively: the crown of the46 tooth is completely decayed below thegum level. Percussion causes minor painreaction. It is planned to extract the 46tooth. The extraction procedure shouldbe performed in:

Page 22: Krok 2 - 2011 Question Paper (Stomatology)

Kok 3 Stomatology 2011 22

A. Haematological departmentB. Stomatological clinicC. Maxillofacial surgery department afterthe consultation with a haematologistD. Stomatological clinic and followingadministration of haemostatic drugsE. Maxillofacial surgery department

153. A 22-year-old patient undergoestreatment for acute deep caries of the 26thtooth. The pulp-capping material of choi-ce is Ca(OH)2/salicylate cement "Life",the constant filling material is ligh-curecondensable composite "Solitaire-2". Thepulp cap should be covered with:

A. Glass-ionomer cementB. Adhesive composite systemC. Insulating varnishD. Phosphate cementE. Zinc-eugenol cement

154. A group of specialists conducts anepidemiological survey of certain agegroups of population aimed at evaluati-on of periodontal disease prevalence andtreatment needs. These rates are studiedby means of the following index:

A. CPITN (WHO index)B. OHI-S (Green-Vermillion index)C. PDI (Ramfjord index)D. PI (Russel index)E. PMA (Parma)

155. A 27-year-old victim was delivered toto the facial surgery department with acut wound of right cheek. The trauma iscivilian, the patient got it in a fight. Thedeadline for early surgical d-bridementwithout preliminary antibiotic injection isno more than:

A. 24 hoursB. 48 hoursC. 72 hoursD. 12 hoursE. 1 hour

156. A 28-year-old patient has beenadmitted to the oral and maxillofacialsurgery for a carbuncle of face. Objecti-vely: the patient is in a critical conditi-on, face, mucous membranes of lips andconjunctiva are edematous and cyanotic.Body temperature is 40oC. The patientpresents with delirium, hallucinations.The carbuncle is located on the upper lip,the surrounding soft tissues are swollen.Palpebral fissures are closed due to theedema of eyelids. What is the most likelycomplication of carbuncle?

A. Cavernous sinus thrombosisB. MeningitisC. Cerebral abscessD. SepsisE. Orbit phlegmon

157. A 37-year-old patient has beendiagnosed with abscessed furuncle ofthe upper lip. After dissection of theinflammation focus the exudate has beensent for bacteriological analysis. Whichcausative agent for furuncle is most likelyto be found?

A. Staphylococcus monocultureB. Colibacillus monocultureC. Proteus monocultureD. Staphylococcus and protozoa associati-onE. Streptococcus monoculture

158. A 54-year-old patient complains ofa tumor-like formation in the region ofhis lower lip which appeared 1-1,5 monthsago. Objectively: there is a round tumour-like formation up to 1cm in diameteron the red border of the lower lip onthe right. Roll-like edges of the formati-on protrude above the level of the redborder. In the centre of the formation abrown crust is visible. After its removal acrateriform hole can be seen. There is nobleeding or pain. Regional lymph nodesare not enlarged. What is the most likelydiagnosis?

A. KeratoacanthomaB. Lower lip carcinomaC. Lower lip papillomaD. Lower lip fibromaE. Verrucous carcinoma

159. A 26-year-old patient presented ata clinic for prosthetics. Objectively: thecrown of the 16 tooth is destroyed by1/3. It is planned to restore its anatomi-cal shape with a metal inlay. What is thefirst stage of cavity preparation?

A. Cavity widening and necrosectomyB. Making additional cavitiesC. BevelingD. Completing the cavity floorE. Completing the cavity walls

160. A 52-year-old female patientcomplains of severe paroxysmal pain inthe region of the 34 tooth. Throughoutthe year the 35, 36, 37 teeth were sequenti-ally extracted because of pain complaints.Percussion of the 34 tooth causes acuteparoxysmal pain. The same pain arises

Page 23: Krok 2 - 2011 Question Paper (Stomatology)

Kok 3 Stomatology 2011 23

when talking and washing the projectionarea of the mental foramen. Clinical androentgenologic examination revealed nosigns of bone tissue destruction. What isthe most likely diagnosis?

A. Neuralgia of the 3 branch of trigeminusB. 34 tooth pulpitisC. Facial nerve neuritisD. Ganglionitis of pterygopalatine gangli-onE. Neuritis of the 3 branch of trigeminus

161. A 45-year-old female patientcomplains of a slightly painful, tumour-like formation in the right parotid regi-on that appeared two months ago forno apparent reason. The patient notesweakness, occasional body temperaturerise up to 37, 2oC − 37, 6oC. Two weeksago she underwent the anti-inflammatorytherapy which did not have positive effect.Objectively: the face is asymmetric dueto a minor swelling in the right parotidregion. Palpation reveals a roundish denseand elastic formation with even contours,it is slightly painful, not adherent to theskin. Incisive canal exudes a small amountof transparent saliva. What is the most li-kely diagnosis?

A. Hertsenberg’s pseudoparotitisB. Parotid atheromaC. Acute infectious parotitisD. Epidemic parotitisE. Sjogren’s syndrome

162. An 8,5-year-old child is apparentlyhealthy. The child complains about painin an upper tooth on the left caused bytraumatic injury sustained three hoursago. Objectively: the crown part of the 21tooth is destroyed by 1/2, the pulp is redand significantly exposed, probing causesacute pain and bleeding. Percussion of the21 tooth is extremely painful. Choose themost efficient treatment method of the 21tooth:

A. Vital amputationB. Vital extirpationC. Devital amputationD. Devital extirpationE. Bioassay technique

163. A 23-year-old patient complai-ns about root exposition, gingivalhemorrhage during tooth brushing, gumitch. Objectively: there is supragingivaland subgingival dental calculus. Gums arehyperaemic, edematic, pouches are 3,5mm deep. X-ray picture shows resorpti-on of interalveolar septa by 1/3. What is

the most likely diagnosis?

A. Chronic generalized I degree peri-odontitisB. Chronic generalized II degree peri-odontitisC. Exacerbation of generalized I degreeperiodontitisD. Exacerbation of generalized II degreeperiodontitisE. II degree periodontitis

164. A 14-year-old child complains aboutacute spontaneous spasmodic pain in anupper jaw tooth on the right. The painhas been lasting for 3 days, it is throbbing,irradiating to the temple, getting worse atnight. Objectively: surface of the 15 toothexhibits a carious cavity within parapulpardentine. Dentine is softened, of greyishcolour. Probing of the whole cavity floor ispainful, percussion of the 15 tooth is pai-nless. What is the most likely diagnosis?

A. Acute purulent pulpitisB. Acute diffuse pulpitisC. Acute focal pulpitisD. Acute periodontitisE. Exacerbation of chronic periodontitis

165. A 12-year-old boy complains ofsevere pain caused by cold food in the11 tooth. A day before he had a crownbreak off of the 11 tooth. Examinati-on revealed an oblique fracture of the11 tooth crown with the opening of thetooth cavity, probing causes acute pain,comparative percussion is painful, thereis minor tooth mobility. What treatmentshould be administered?

A. Vital extirpationB. Devital extirpationC. Biologic methodD. Vital amputationE. Devital amputation

166. A gravida (36 weeks) complainsof gingival haemorrhages and excessi-ve plaque despite a thorough hygieniccare of oral cavity. Objectively: gingi-val papilla and marginal gingival edgeare spongiose, bleeding when touched.Fedorov-Volodkina’s hygiene indexis 3,7points. What kind of toothpaste shouldbe recommended to this patient after theprofessional oral hygiene?

Page 24: Krok 2 - 2011 Question Paper (Stomatology)

Kok 3 Stomatology 2011 24

A. Chlorhexidine-containing toothpasteB. Toothpaste with mineralizing agentsC. Herbal toothpasteD. Fluorine-containing toothpasteE. Gel toothpaste

167. A girl is 13 years old. She lives in anarea where fluoride concentration in thedrinking water is at the rate of 1,6 mg/l.Dental examination revealed some chalk-like spots on the vestibular surfaces of allher teeth. The white coloration is moreintense in the cenre and less on the peri-phery. There is light-brown pigmentationin the region of the central incisors alongthe cutting edge. What is the most likelydiagnosis?

A. Dental fluorosisB. Acute initial cariesC. Systemic enamel hypoplasiaD. Stainton-Capdepont syndromeE. Amelogenesis imperfecta

168. An 8-year-old child is found to haveconvex facil profile, forced closing of li-ps, sagittal gap of 7 mm. Eschler-Bittnertest produces some face improvement.This abnormality can be eliminated bymeans of Frankel type regulator. What isthe mechanism of action of this device?

A. Normalization of labial, buccal andlingual pressure as well as of mandiblepositionB. Inhibition of maxilla growth in thesagittal directionC. Maxillary expansion by means of ascrewD. Normalization of mandible positionand growth by means of intermandibulartractionE. Normalization of upper front teethposition by means of a vestibular bar

169. A 40-year-old patient has been givenanesthesia, immediately after which hedeveloped severe weakness, heart palpi-tation. Objectively: the patient is consci-ous, inert, skin is markedly hyperemic,there is acute headache, AP is 180/100 mmHg, heart sounds are muffled. What stateis it?

A. Hypertensic crisisB. Cardiogenic form of anaphylactic shockC. An attack of stenocardiaD. Cardial painE. Myocardial infarction

170. A completely edentulous 70-year-oldpatient has ordered complete removabledentures. Artificial teeth are placed upon

the spherical surface. Close teeth contactby movements of the lower jaw will beguaranteed by the following average radi-us of the spherical surface:

A. 9 cmB. 5 cmC. 7 cmD. 12 cmE. 18 cm

171. A 13 year-old girl has presentedwith gingival haemorrhage and mobi-lity of the front teeth over the lastmonth. Objectively: gingival mucosaaround the lower incisors and cani-nes is edematous, hyperemic, bleeds ontouch. There is grade l tooth mobili-ty, periodontal pockets are 3 mm deep.Orthopantomogram shows the resorptionof interalveolar septa by 1/3 of their hei-ght. There is crowding of the lower frontteeth. Hygiene index is 4,2. The patienthas to consult the following specialist:

A. OrthodontistB. NeuropathologistC. EndocrinologistD. HaematologistE. Gastroenterologist

172. A 13-year-old girl has been experi-encing gingival hemorrhages and frontalteeth mobility throughout the last month.Objectively: gingival mucous membranein the region of lower incisors and caninesis edematic, hyperemic, it bleeds on touch.These teeth present with I degree mobili-ty, parodontal pouches are 3 mm deep.Orthopantomogram shows reduction ofbone tissue of interalveolar septa by 1/3of their height. Frontal teeth of the lowerjaw are overcrowded. Hygiene index is4,2. What local treatment should be provi-ded in the first place?

A. Professional hygieneB. Anti-inflammatory therapyC. Orthodontic careD. Physical therapyE. Antibacterial therapy

173. A 65-year-old patient complains of abreak off of three front artificial porcelainteeth of the removable denture fabri-cated 2 months ago. In what way are theporcelain teeth linked with the plasticdenture base?

Page 25: Krok 2 - 2011 Question Paper (Stomatology)

Kok 3 Stomatology 2011 25

A. By means of crampon-fixing wireelementsB. By means of glueC. Due to physicochemical connectionD. By means of claspsE. By means of diatoric cavities

174. A 47-year-old patient complains ofpersistent ache in the 27 tooth, thatgets worse when biting. Objectively: skincolour is normal, the face is symmetrical,the mouth opens without any difficulties,alveolar mucosa at the level of 27 tooth isedematous and hyperemic. The 27 toothpresents a deep carious cavity communi-cating with the pulp chamber. Percussionof the 27 tooth causes acute pain. What isyour provisional diagnosis?

A. Exacerbation of acute periodontitis ofthe 27 toothB. Acute purulent maxillary periostitis inthe region of the 27 toothC. Acute purulent periodontitis of the 27toohD. Acute generalized purulent pulpitis ofthe 27 toothE. Chronic odontogenic left-sided maxi-llary sinusitis

175. A patient complains of "clicking"inthe TMJ. On auscultation, the sounds canbe heard in the middle and at the begi-nning of mouth opening. These signs giveevidence of:

A. Reduction in occlusal vertical dimensi-onB. Increase in occlusal vertical dimensionC. TMJ dislocationD. Joint capsule distensionE. Joint subluxation

176. A 32-year-old patient presented to aclinic for takingof an upper jaw impressi-on. During this procedure she choked wi-th a piece of impression material, therewas risk of asphyxia. What are the first-priority actions within emergency care?

A. Abrupt push-like compressions of thelower thoraxB. Artificial ventilation of lungsC. Plaster removal from the airwaysD. Closed-chest cardiac massageE. Tracheotomy

177. A 32-year-old 3-month pregnant pati-ent presented to the prosthodontics cli-nic for the prosthetics. Objectively: the 16tooth is missing. It is planned to fabri-cate a dental bridge supported by the15 and 17 teeth. The patient was gi-

ven an anaesthetic of articaine group”Ultracain DS”. The orthopaedist chosethis anaesthetic because:

A. It is low-toxic, is not capable of crossingthe hemato-placental barrierB. It is low-toxicC. It reduces the volume of uteroplacentalcirculationD. It is most commonly used in thestomatological practiceE. It doesn’t contain sulphites

178. A patient consulted a dental surgeonabout extraction of the roots of the36 tooth. The tooth decayed long ago.Objectively: the crown of the 36 isdestroyed by 2/3, percussion is painless,mucosa around the 36 tooth exhibits noevident pathological changes. X-ray pi-cture of the alveolar process shows aroundish well-defined radiolucency nearthe apex of the medial root. What is themost likely diagnosis?

A. Chronic granulomatous periodontitisB. Exacerbation of chronic periodontitisC. Chronic granulating periodontitisD. Acute purulent periodontitisE. Chronic fibrous periodontitis

179. A 52-year-old patient complai-ns about teeth mobility, gingivalhaemorrhage. The 44, 45, 34, 35 teethpresent with I degree mobility, the 43,42, 41, 31, 32, 33 teeth present with IIdegree of teeth mobility. There is gingivaledema, parodontal pouches are 5-6 mmdeep. X-ray picture shows destruction ofbone tissue by 1/2 of root length. A dentistmade a diagnosis: generalized periodonti-tis of median severity. What orthopaedicconstruction is the most efficient in thissituation?

A. Elbrecht splintB. Plastic mouthguardC. Murray’s splintD. Mamlock splintE. Temporary Novotny splint

180. A victim got a face and temporal regi-on trauma. A doctor made a diagnosis:fracture of malar arch. What processes ofcranial bones are injured?

Page 26: Krok 2 - 2011 Question Paper (Stomatology)

Kok 3 Stomatology 2011 26

A. Temporal process of zygomatic boneand zygomatic process of temporal boneB. Zygomatic process of frontal bone andzygomatic process of temporal boneC. Temporal process of zygomatic boneand zygomatic process of frontal boneD. Zygomatic process of maxilla andzygomatic process of temporal boneE. Zygomatic process of maxilla andzygomatic process of frontal bone

181. A 7-year-old child sustained a sportinjury. He complains about pain in theregion of mandible, inability to join histeeth. Pressing against the chin causesintensification of pain in the region ofmandible on the right. The child is di-agnosed with a fracture of mandible bodywithout fragment displacement. Whattherapeutic tactics should be chosen inorder to prevent the displacement of bonefragments in the transitional dentition?

A. Fragment immobilization by means ofan intraoral splintB. Administration of anti-inflammatorytherapyC. Application of sling bandageD. Fixation by means of Tigerstedt splintsE. Osteosynthesis

182. At the end of the reception hoursa dental surgeon has to fill in thestomatological forms. Name the form ofeveryday reporting on children dentalreception:

A. Form 39B. Form 9C. Form 19D. Form 29E. Form 49

183. A patient has orthopaedic indicationsfor the 37 tooth extraction. What kind ofanaesthesia should be given to this patientbefore the extraction?

A. Mandibular torus anaesthesiaB. Infraorbital + buccal anaesthesiaC. Mental + buccal anaesthesiaD. Tuberal + buccal anaesthesiaE. Mandibular

184. On removing a dental plaque, a 19-year-old patient is found to have two whi-te patches in the precervical region of the11 and 21 teeth. After drying the patchesincrease in size. What is the most likelydiagnosis?

A. Initial cariesB. FluorosisC. Local hypoplasiaD. Enamel necrosisE. Amelogenesis imperfecta

185. A 27-year-old patient has beenreferred for endodontic treatment on the45 tooth. Objectively: the tooth crown isdecayed, the lateral surface of tongue andcheek mucosa being in direct contact wi-th the 45 tooth are covered with grayi-sh plaques of macerated epithelium thatare somewhat raised above the surface ofthe mucous membrane. Uvula and palati-ne arches are of dark red colour, andthe hard palate has papules with a redrim and covered with grayish epithelium.Submandibular, cervical, supraclavicular,subclavicular lymph nodes are enlarged,painless. What is your provisional di-agnosis?

A. Secondary syphilisB. Chronic recurrent aphthous stomatitisC. Lupus erythematosus, plaque stageD. White sponge nevusE. Lichen ruber planus

186. Examination of a 6-year-old boyrevealed enlarged lymph nodes in bothsubmandibular and cervical regions.Objectively: the 75, 84 and 85 teethare decayed, there are presentations ofcommissural cheilitis. According to theboy’s mother, he has been quickly getti-ng tired, sweating from the least physi-cal exercise, complaining about weaknessthroughout the last 2-3 months. He alsogave up training in a sports class. Whatplan of additional examination should beadopted?

A. Complete blood count, haematologistconsultationB. Biochemical blood test, endocrinologistconsultationC. Puncture biopsy, oncologist consultationD. Magnetic resonance tomography,immunologist consultationE. Pulmonary roentgenography,pulmonologist consultation

187. A child was born with schistasis ofalveolar process, hard and soft palate. Theoptimal way to feed the child before thesurgery will be through:

Page 27: Krok 2 - 2011 Question Paper (Stomatology)

Kok 3 Stomatology 2011 27

A. ObturatorB. Enteric feeding tubeC. Baby bottle nippleD. SpoonE. -

188. A 32-year-old patient complainsabout pain and swelling in the region ofhis mandible and left cheek. He has beensuffering from this for 2 days. Objecti-vely: his general condition is satisfactory.Body temperature is 37, 5oC. There is anedema of soft tissues in the left buccalregion. Submandibular lymph nodes arebilaterally enlarged, painful on palpati-on. Crown of the 37 tooth is partiallydecayed, the tooth is immobile, percussi-on is slightly painful. Mucogingival foldin the region of the 37 tooth is edematic,hyperaemic, flattened, its palpation causesacute pain. What is the most likely di-agnosis?

A. Acute purulent periostitis of mandiblebeginning from the 37 toothB. Acute odontogenous osteomyelitis ofmandibleC. Chronic productive periostitis of mandi-ble beginning from the 37 toothD. Chronic odontogenous osteomyelitis ofmandibleE. Abscess of alveololingual groove on theleft

189. A 7-year-old child presents withf faci-al asymmetry, severe pain in the lowerjaw on the left, body temperature riseup to 39, 0oC. Objectively: the face isasymmetric due to the massive swellingof soft tissues in the mandibular region onthe left. The skin over the infiltration ishyperemic and cannot be plicated. Mouthopening is limited, painful. The crown ofthe 75 tooth is destroyed by 2/3, the toothexhibits the grade II mobility. Percussionof the 36 tooth is painful, the 75 tooth -painless. Edema and fluctuation of soft ti-ssues on the buccal and lingual sides ofthese teeth are present. What measuresare to be taken in the first place?

A. To extract the 75 tooth, to lance theinflammation focus on both sidesB. To extract the 75 tooth, to lance theinflammation focus on the buccal sideC. To extract the 75, 36 teeth, to lance theinflammation focus on the buccal sideD. To lance the inflammation focus on thelingual and buccal sidesE. To lance the inflammation focus in thesubmandibular region

190. A 42-year-old patient complains ofa cavity in the 26 tooth. Objectively: the26 tooth exhibits a deep carious cavity,dentine is dense, pigmented, there is painreaction to a cold stimulus, percussion andprobing are painless. What is your provi-sional diagnosis?

A. Chronic deep cariesB. Acute deep cariesC. Chronic fibrous pulpitisD. Chronic median cariesE. Acute median caries

191. A 12-year-old boy repeatedly visi-ts a dentist to finish treatment of the36 tooth for the exacerbation of chronicperiodontitis. The complaints are absent.Objectively: occlusive dressing on the 36tooth is preserved, tooth percussion is pai-nlessl, mucous membrane of the 36 hasno pathological changes, is painless onpalpation. What material should be usedfor root canal filling in this case?

A. Sealer with a gutta-percha pointB. Zinc-eugenol pasteC. Resorcinol-formalin pasteD. Calcium-containing pasteE. Phosphate cement

192. A 63-year-old patient requires a full-arch denture for the upper jaw. Objecti-vely: the 18, 17, 16, 15, 25, 26, 27 teethare missing, the gag reflex is not marked,the torus is of average size. In order toavoid possible inconvenience while usingthe denture, the arch should be located atthe following distance from the palatinefossae:

A. 10-12 mmB. 4-8 mmC. 23-25 mmD. 2-3 mmE. 16-22 mm

193. A 3-year-old patient presented tothe prosthetic dentistry clinic for thetraumatic loss of an upper central incisor.The patient hasn’t undergone prostheti-cs before. He has a history of asthma.What are the first-priority measures ofbronchospasm prevention?

A. Tests for plastic and metal allergyB. Preparation following the anaestheti-zationC. Teeth treatment with a remineralizingsolutionD. Fabrication of a temporary constructionE. Administration of sedative drugs

Page 28: Krok 2 - 2011 Question Paper (Stomatology)

Kok 3 Stomatology 2011 28

194. A 47-year-old patient complains ofpain in the lateral part of mandible to theleft of the toungue edge, which is causedby tongue movements. Two days ago thepatient was provided with claspps’ fixationa partial removable clasp denture for the47, 42, 33 teeth. Retroalveolar region ofthe mucous membraneis exhibits a locali-zed hyperemic area matching the edge ofthe denture. Which muscle contractionsdemarcate the boundary in this region?

A. MandibulohyoidB. GeniohyoidC. HyoglossalD. Medial wing muscleE. Masticatory

195. A 40-year-old female patientcomplains about crepitation and pain inthe region of TMJ projection on the ri-ght, that arise in the final stage of mouthopening. Articular sounds turned up afterdental prosthetics. Mouth opening is unli-mited, mouth is opened in a straight path,amplitude of mouth opening is 5 cm. Whatis the most likely diagnosis?

A. Lower jaw subluxationB. Lower jaw dislocationC. Anterior dislocation of articular diskD. Posterior dislocation of articular diskE. Meniscus tresis

196. A 40-year-old patient needs the oralcavity sanation, which involves extractionof the upper central incisors. What nervestructure will be blocked as a result of inci-sor anaesthesia?

A. Nasopalatine nerveB. Anterior superior alveolar branch ofthe infraorbital nerveC. Median superior alveolar branch of theinfraorbital nerveD. Posterior superior alveolar branch ofthe infraorbital nerveE. Dental nerve plexus

197. A maxillofacial surgery departmentadmitted a patient with a half-openmouth, his chin was put forward and devi-ated to the right. On palpation, the leftmandibular fossa was found to be empty.Internal palpation failed to detect the left

articular head. Mandibular movementswere possible but restricted. What is themost likely diagnosis?

A. Unilateral dislocation of the left TMJB. Unilateral dislocation of the right TMJC. Bilateral dislocation of the TMJD. Acute arthritis of the left TMJE. Acute arthritis of the right TMJ

198. A 68-year-old patient withcompensated insulin-dependent diabetesmellitus had to spend 8 hours at a hospi-tal. Within this time he didn’t eat and wasnervous. The patient became aggressi-ve, pale and sweaty, then he lost consci-ousness. What is the most likely di-agnosis?

A. Hypoglycemic comaB. Anaphylactic shockC. Hypertensic crisisD. Acute respiratory failureE. Collapse

199. Rising from a chair after a dentalprocedure, a 60-year-old patient feltretrosternal pain radiating to the left arm,nausea, weakness. Objectively: the patientis pale, his face is sweaty. AP is 90/60 mmHg, pulse is arrhythmic, of poor volume,100/min. Heart sounds are muffled, vesi-cular breathing is present. What is yourprovisional diagnosis?

A. An attack of coronary artery diseaseB. Myocardial infarctionC. Hypertensic crisisD. StenocardiaE. Pulmonary artery thrombosis

200. During the cystectomy involvingresection of the root apices of the 11,21 and 22 teeth a 45-year-old patientdeveloped skin pallor and cyanosis, a coldsweat, a sharp drop in blood pressure(70/40 mmHg). The patient is consciousbut complains of weakness. What statehas been developed?

A. CollapseB. HaematomaC. Epileptic seizureD. Quincke’s edemaE. Syncope